Contracts

Réussis tes devoirs et examens dès maintenant avec Quizwiz!

Which of the following service contracts must satisfy the Statute of Frauds to be enforceable? response - incorrect Incorrect A A contract for a specific task that will take approximately 12 months to complete Correct B A contract for one month of service that is to begin 13 months in the future C A contract for the lifetime of the client D A contract for the client's personal care during an illness of unknown duration

A contract that by its terms cannot be performed within one year is subject to the Statute of Frauds. The date runs from the date of the agreement and not from the date of performance. Thus, a contract for one month of service that is to begin 13 months in the future must satisfy the Statute to be enforceable. If the contract is possible to complete within one year, it is not within the one-year prong of the Statute of Frauds, even though actual performance may extend beyond the one-year period. A specific task that will take approximately 12 months to complete might be completed in less time. Likewise, a contract for the client's personal care during an illness of unknown duration might be completed in less than a year if the client recovers quickly. A contract for the lifetime of the client is not within the Statute because it is capable of performance within a year since the client could die at any time.

A photography buff wrote a letter to his brother-in-law offering to sell him his camera for $1,500, because he knew that he had admired it for a long time. The day after the brother-in-law received the letter, he mailed a letter back to the photography buff agreeing to purchase the camera equipment for $1,500. The next day, after describing the camera to a friend who was very knowledgeable about photographic equipment, the brother-in-law learned that the camera was second-rate and not worth more than $1,200. He immediately telephoned the photography buff and told him that he had no interest in buying the camera. The photography buff received his brother-in-law's letter agreeing to purchase the camera equipment a day after receiving the phone call. If the photography buff brings an action against his brother-in-law for breach of contract, and the brother-in-law defends on the grounds that no contract was formed, how should the court rule? response - correct AFor the brother-in-law, because the description of the subject matter of the contract was too indefinite to be enforced. BFor the brother-in-law, because the photography buff received the telephone call before he received the letter. Correct CFor the photography buff, because his brother-in-law's letter accepting the offer was effective when mailed. DFor the photography buff, because the contract is for the sale of goods over $500 in value and his brother-in-law's attempted rejection of the offer was oral.

A contract was formed because the brother-in-law's acceptance was effective on dispatch. Under the "mailbox rule," acceptance by mail or similar means creates a contract at the moment of posting, properly addressed and stamped, unless: (i) the offer stipulates that acceptance is not effective until received; or (ii) an option contract is involved. Here, the brother-in-law dispatched first an acceptance and then a rejection of the photography buff's offer. The mailbox rule applies because the photography buff's offer did not specify that acceptance was not effective until receipt, nor is an option contract involved. Because the brother-in-law dispatched his acceptance before he called with his rejection, the mailbox rule applies. Thus, the brother-in-law's acceptance was effective, thereby creating a contract at the moment it was mailed, and his attempted rejection was ineffective. (B) is incorrect because once the acceptance was effective, the fact that the photography buff received the "rejection" by telephone before he received the acceptance letter has no effect on the formation of the contract. (A) is incorrect because the letter from the photography buff indicates that the subject matter of the contract was his camera that the brother-in-law had admired for some time. This description on its face appears to be sufficiently definite that a court would be able to determine with reasonable accuracy which camera is subject to the photography buff's offer to sell. (D) is incorrect even though it is true that, pursuant to the Statute of Frauds, a contract for the sale of goods of $500 or more is not enforceable unless evidenced by a writing. There is no requirement that a rejection of an offer to enter into such a contract must be in writing.

When a contractor is under a contractual duty to construct a building and the building is destroyed by an act of nature while it is still a work in progress, the destruction __________. response - correct A will discharge the contractor's duty to perform Correct B will not discharge the contractor's duty to perform, but will extend the date of performance C will discharge the contractor's duty to perform if rebuilding cannot be reasonably completed by the date of performance D will neither discharge the contractor's duty to perform nor extend the date of performance

A contractor's duty to construct a building is not discharged by destruction of the work in progress. However, if the destruction was not caused by the contractor, such as by an act of nature, most courts will extend the date of performance beyond the original deadline.

When a party that offers a rare or unique service has breached a service contract, the court may grant __________ to the nonbreaching party. response - correct A specific performance B punitive damages C restitution Correct D injunctive relief

A court may grant injunctive relief to enjoin a breaching party from working for a competitor throughout the duration of the contract if the services contracted for are rare or unique. Specific performance is not available for breach of a contract to provide services, even if the services are rare or unique because it is difficult to supervise and most courts find that it is tantamount to involuntary servitude. Punitive damages are generally not awarded in contract cases. Restitution is available as an alternative to contract damages in a non-contract situation to prevent unjust enrichment. Here there is a contract and there are no facts indicating unjust enrichment.

A letter of revocation of an offer becomes effective at the moment it is __________. response - correct A Dispatched by the offeror Correct B Received by the offeree C Read by the offeree D Signed by the offeror

A letter of revocation of an offer is effective when it is received by the offeree. At common law, a written communication is considered to have been "received" when it comes into the possession of the person addressed (or of someone authorized by him to receive it) or when it is deposited in some place authorized as the place for this or similar communications to be deposited. Similarly, under the UCC, a person receives notice when it comes to his attention, or it is delivered at a place of business through which the contract was made or another location held out by that person as the place for receipt of such communications. An organization receives a communication at the time it is brought (or should have been brought) to the attention of the individual conducting the transaction. Thus, the written revocation need not be actually read by the offeree to be effective. A revocation generally is effective when received while, in contrast, an acceptance generally is effective when dispatched (i.e., the mailbox rule). Under the mailbox rule, if the offeree dispatches an acceptance before he receives a revocation sent by the offeror, a contract is formed.

A man and a woman met in a bar. While the two enjoyed a couple of drinks, the woman told the man that she greatly admired the diamond stickpin he had in his lapel. "Oh, this," the man laughed. "It's no diamond; it's only a piece of glass." The woman acknowledged his statement, but kept commenting on how nice it looked. After further conversation, the man orally agreed to sell the stickpin to her for $500. They agreed that in four days, the man would bring the stickpin to the same bar, and the woman would bring the $500 in cash. The woman wrote down her name and phone number on a napkin and asked the man to call her if there were any change in plans. The man duly appeared with the pin, but the woman failed to appear. The man filed suit against the woman for $500. In an action by the man against the woman for breach of contract, which of the following would be the woman's best defense? response - correct A$500 was an unconscionable amount to pay for a piece of glass. BThe parties lacked capacity to contract because they were drinking alcohol. Correct CThe agreement violated the Statute of Frauds. DNeither the woman nor the man was a merchant.

A promise for the sale of goods priced at $500 or more is not enforceable under the Statute of Frauds unless evidenced by a writing signed by the party to be charged. Here, the woman is the party to be charged, and her promise to pay $500 was only oral. The napkin with the woman's name and number would not be sufficient to satisfy the Statute. To satisfy the Statute of Frauds, the UCC requires that the writing indicate that a contract has been made and specify the quantity term. Here, the napkin does not indicate that there is a contract. It merely contains a name and phone number. A court could not enforce a promise based on the writing. Thus, the woman's promise is unenforceable. (A) is incorrect because the concept of unconscionability allows avoidance of a contract only where the terms are so one-sided as to indicate unfair surprise or a contract of adhesion. Here, neither party had superior bargaining power, and the woman knew exactly what she was buying when she made the agreement. (B) is incorrect because just having alcoholic drinks does not mean that parties no longer have the capacity to contract. To lack capacity due to intoxication, a party must be so intoxicated that the party does not understand the nature and significance of his promise. Nothing in the facts indicates that these parties were intoxicated, let alone so intoxicated that they did not understand what they were doing. (D) is incorrect because the parties' status as nonmerchants is irrelevant. While the Code relaxes the Statute of Frauds rule in the case of a written confirmation between merchants, that exception does not apply here. There was no writing by either party that would satisfy the Statute of Frauds; thus, even if the parties were merchants, the agreement would be unenforceable.

Which of the following statements regarding revocation and acceptance of contract offers is correct? response - correct A A revocation generally is effective when dispatched, and an acceptance generally is effective when dispatched. Correct B A revocation generally is effective when received, and an acceptance generally is effective when dispatched. C A revocation generally is effective when dispatched, and an acceptance generally is effective when received. D A revocation generally is effective when received, and an acceptance generally is effective when received.

A revocation generally is effective when received and an acceptance generally is effective when dispatched (i.e., the mailbox rule). Under the mailbox rule, if the offeree dispatches an acceptance before he receives a revocation sent by the offeror, a contract is formed.

At common law, a written communication revoking an offer is considered "received" by an offeree at the moment: response - correct Correct A It comes into the offeree's physical possession B The offeror dispatches the communication, provided that the mail is properly addressed and stamped C The offeror relinquishes possession of the communication D The offeree reviews the contents of the revocation

A written revocation of an offer is effective when it is received by the offeree. At common law, a written communication is considered to have been "received" as soon as it comes into the physical possession of the person addressed (or of someone authorized by him to receive it) or when it is deposited in some place authorized as the place for this or similar communications to be deposited. The rule for revocation is different from the rule for acceptance, which generally creates a contract at the moment of dispatch, provided that the mail is properly addressed and stamped. The offeree need not review the contents of the revocation for it to be effective.

Which of the following best states the result when, in a contract between merchants for the sale of goods, the acceptance does not match the terms of the offer? response - correct A Additional terms may be knocked out of the contract, but different terms will be included unless the offeror has already objected to the particular terms. Correct B Additional terms that do not materially alter the original terms of the offer will be included in the contract, but different terms may be knocked out of the contract. C Neither additional terms nor different terms are ever included in the contract. D Additional terms are never included in the contract, but different terms will be included unless the offer expressly limits acceptance to the terms of the offer.

Additional terms that do not materially alter the original terms of the offer will be included in the contract. In a contract between merchants for the sale of goods, additional terms in the acceptance will be included in the contract unless they materially alter the original terms, the offer expressly limits acceptance to the terms of the offer, or the offeror has already objected to the particular terms or objects within a reasonable time. Different terms may be knocked out of the contract. There is a split of authority on different terms. Some states follow the same rules as for additional terms, but others employ the knockout rule. Under the knockout rule, conflicting terms are knocked out of the contract and gaps are filled by the UCC gap-filler provisions. Conflicting terms are subject to the knockout rule; additional terms are never subject to the knockout rule.

The owner of an exclusive clothing salon entered into a written agreement with a customer to sell the customer a certain full-length fake fur coat for $12,000, with delivery by December 7. On December 5, the customer went to the salon at 5:30 p.m., and the salon owner told her that her coat was ready and she could take it home with her. The customer inspected the coat and discovered that a button was missing. She told the salon owner that she would not accept the coat without the missing button. The salon owner informed the customer that his tailor had gone home for the day but he assured her that the coat could be ready with the button sewn on the next morning. Which of the following best states the customer's position? response - correct AThe customer may reject the coat, because the salon owner failed to provide perfect tender. Correct BThe customer may reject the coat, but she must give the salon owner an opportunity to cure. CThe customer must accept the coat, because its value is not substantially impaired by the missing button. DThe customer must accept the coat, because the defect can be easily cured.

Although the customer is entitled to reject the coat for even a minor defect such as one button being missing, she is required to give the seller an opportunity to cure this defect. Because this is a contract for the sale of goods, the Uniform Commercial Code applies. Pursuant to the UCC, if goods or any tender fail in any respect to conform to the contract, the buyer may reject the goods. This rule of perfect tender allows rejection for any defect, and does not require material breach. However, the perfect tender rule is softened by the rules allowing the seller to cure the defect by giving reasonable notice of an intention to cure and making a new tender of conforming goods within the time originally provided for performance. Here, one button missing on a $12,000 coat is a very minor defect. However, pursuant to the perfect tender rule, the customer has the right to reject the coat even for this defect. In turn, the salon owner is entitled to cure the defect by notifying the customer of his intention to do so and by making a conforming tender. The salon owner has told the customer that the tailor will sew on the button by the next day, which will result in the coat's conforming to the contract. At this point the customer must give the salon owner an opportunity to cure. (A) is incorrect because it fails to add that the salon owner must be allowed an opportunity to cure, as discussed above. (C) and (D) are incorrect because with the failure of perfect tender, the customer is not required to accept the coat. This right of rejection is not affected by the minor nature of the defect, as (C) suggests. Similarly, the buyer's right of rejection is not impaired by the fact that the defect is easily curable, as (D) states.

An option contract is a distinct contract in which __________ a promise not to revoke an outstanding offer. response - correct Correct A An offeree gives consideration for B An offeror gives written assurances regarding C A merchant makes D A nonmerchant makes

An offer normally can be revoked at will by the offeror. An option contract is a distinct contract in which an offeree gives consideration for a promise by the offeror not to revoke an outstanding offer. Even if an offeror gives written assurances regarding a promise not to revoke for a certain period, the revocation-at-will rule applies unless the offeror's power to terminate the offer has been limited in some way, such as by the creation of an option contract. Note that if the offeror could reasonably expect that the offeree would rely to her detriment on the written assurances and the offeree does so rely, the offer could be held irrevocable as an option contract, but the assurances alone are generally not enough to make an offer irrevocable. Under the Merchant's Firm Offer rule in Article 2, a promise to keep an offer open will be enforceable without the payment of consideration when a merchant offers to buy or sell goods in a signed writing that gives assurances that the contract will be held open for a specified period of time. This is not the same as an option contract, which is a distinct contract and generally requires that the offeree give some consideration for the promise not to revoke.

A court order requiring a breaching party to perform under the contract or face contempt of court charges is also known as: response - correct A Replevin Correct B Specific performance C Laches D An injunction

An order for specific performance is essentially an order from the court directing the breaching party to perform as promised under the contract or face contempt of court charges. In contrast, an injunction is usually a court order prohibiting someone from doing a specified act. Although a mandatory injunction may order a party to perform a particular act, an order requiring a party to perform under the contract or face contempt is an order for specific performance, not an injunction. Replevin is a buyer's right to replevy (recover) undelivered, identified goods from a seller under certain circumstances. Replevin will lie only in cases with identified goods. Laches is an equitable defense involving an unreasonable lapse of time in asserting a right that prejudices the defendant. It can be raised to defend an action for specific performance; the breaching party would argue that the other party delayed too long in bringing the specific performance action, and the delay caused prejudice against the breaching party.

Which of the following statements is correct? response - correct A Both an ordinary option contract and a merchant's firm offer require that the offeree give consideration Correct B An ordinary option contract requires that the offeree give consideration, whereas a merchant's firm offer does not C Neither an ordinary option contract nor a merchant's firm offer requires that the offeree give consideration D A merchant's firm offer requires that the offeree give consideration, whereas an ordinary option contract does not

An ordinary option contract is a distinct contract in which the offeree gives consideration for a promise by the offeror not to revoke an outstanding offer. In contrast, under Article 2's merchant's firm offer provision, there are circumstances in which a promise to keep an offer open is enforceable even if no consideration has been paid to keep the offer open: A merchant's firm offer arises when a merchant offers to buy or sell goods in a signed writing and the writing gives assurances that the offer will be held open.

On April 15, a wholesaler of tulip bulbs telephoned a local nursery and offered to sell to the nursery 80 gross of tulip bulbs for $8,000, not including delivery charges. The nursery accepted immediately. On April 17, the nursery sent the wholesaler an email confirming the deal for the sale of 80 gross of tulip bulbs for $8,000, and stating that it anticipated a waiver of the delivery charges because of the size of the order. On May 3, the wholesaler telephoned the nursery and stated that, due to a poor growing season for tulips, it would not be able to supply any tulip bulbs to the nursery. If the nursery brings suit against the wholesaler and the wholesaler asserts the Statute of Frauds as a defense, will the nursery prevail? response - correct Correct AYes, because its April 17 email contained the quantity term. BYes, because its April 17 email contained the price term. CNo, because the nursery's April 17 email varied the terms of the wholesaler's offer. DNo, because the wholesaler is the party to be charged and has signed nothing.

Because the quantity was stated in the April 17 email, the Statute of Frauds is satisfied and the nursery may prevail. This contract is for the purchase and sale of goods; thus, the UCC applies. The Statute of Frauds requires that a contract for the sale of goods for $500 or more be evidenced by a writing signed by the party to be charged. This writing must contain the essential elements of the agreement. The quantity term is the key to the sufficiency of a memorandum, and here the writing includes the quantity term. Thus, the writing complied with the Statute of Frauds. (B) is wrong because all other terms (including price) may be proved by parol evidence. The UCC requires only that the memorandum contain (i) quantity, (ii) the signature of the party to be charged, and (iii) a writing sufficient to indicate that a contract was formed. [UCC §2-201] (C) is wrong because it does not bear on the Statute of Frauds issue, but rather on the issue of the additional terms, which will not prevent a contract from being formed between merchants. (D) is wrong because UCC section 2-201(2) provides that, in a deal between merchants, a writing confirming the deal sent by one party will bind both parties, unless the other party objects in writing within 10 days. Here, the wholesaler did not object within 10 days, and so the nursery's email confirmed the deal. Thus, the wholesaler can be charged even though the wholesaler has not signed the memorandum.

A distributor of electric toy trains and a hobby shop owner entered into a written contract providing that the distributor will tender to the shop owner four dozen of a popular electric train set at a price of $100 apiece, to be delivered no later than October 31, to take advantage of the holiday shopping season. The shop owner chose to order from this distributor because its price for the train set was lower than that of other distributors. Shortly after the shop owner placed his order, the distributor raised its prices due to a sudden surge in popularity of that train set. Because the distributor did not have enough train sets to accommodate everyone due to the surge of orders, it decided to deliver train sets only to those buyers who had ordered them at the increased price. The distributor notified the shop owner that it would not deliver the train sets it ordered. The shop owner filed an action to force the distributor to deliver the train sets at the agreed-upon price. Will the court compel the distributor to deliver the train sets to the shop owner? response - correct ANo, because a contract for the sale of goods is not subject to specific performance. Correct BNo, because the shop owner can buy them from another distributor. CYes, because the shop owner will not be able to buy them from another source at the contract price. DYes, because time is of the essence.

Because the shop owner can cover (i.e., buy the train sets from another source), a court will not grant specific performance. If the seller fails to deliver goods under a valid contract, the buyer has a number of remedies available, including the right to cover and the right to obtain specific performance if appropriate. A buyer may obtain specific performance of a contract for the sale of goods if the goods are unique or in short supply, but that does not appear to be the case here because the other distributors carried that train set. Thus, the shop owner can buy the train sets from another distributor and get the difference between the cost of the substitute goods and the contract price. Thus, (B) is correct. (A) is incorrect because, as discussed above, under certain circumstances a seller of goods may be subject to specific performance. (C) is incorrect because, as discussed above, the shop owner can buy the train sets from another distributor and then sue for damages for the difference in cost. Thus, specific performance is not the appropriate remedy. (D) is irrelevant to whether specific performance is granted and is unsupported by the facts.

How can one avoid the preexisting legal duty rule? response - correct A By full performance of the duty Correct B By modifying the original consideration slightly C By making a brand-new identical promise D By beginning performance

Courts are anxious to avoid the preexisting duty rule, which states that the promise to perform, or the performance of, an existing legal duty is not consideration. Thus modifying the original consideration, even slightly, is generally enough to avoid the rule. Making a brand-new identical promise is not sufficient because there is no consideration for the new promise. There must be new consideration or the consideration that is different in some way, such as by accelerating performance, to avoid the preexisting duty rule. Beginning performance does not avoid the preexisting legal duty rule. Even full performance of a preexisting legal duty is not sufficient consideration. There must be some new or different obligation.

A seller of land under an oral sale contract will succeed in a suit against the buyer for the purchase price if: response - correct Correct A The seller conveys the property to the buyer B The buyer is in possession of the property C The seller can prove the agreed-upon purchase price D The buyer has made valuable improvements to the property

Generally, under the Statute of Frauds, a promise creating an interest in land must be evidenced by a writing to be enforceable. A seller can enforce an oral land sale contract only if the seller conveys the property to the buyer. The question asks about enforcement by a seller, and two choices provide elements necessary for enforcement by a buyer. A buyer can specifically enforce a seller's oral promise to sell land if the part performance doctrine applies. Under the doctrine of part performance, conduct that unequivocally indicates that the parties have contracted for the sale of the land will take the contract out of the Statute of Frauds. Most jurisdictions require at least two of the following three actions by the buyer to find part performance: payment, possession, and/or valuable improvements. Proof of the agreed-upon purchase price does nothing to remove an agreement from the Statute of Frauds.

In a contract for a sale of goods priced at $500 or more, if the goods are _________ or __________, the contract will be enforced even if there is no writing. response - correct Correct A Received and accepted; paid for B Shipped; received and accepted C Shipped; paid for D To be specially manufactured; unique

If goods are either received and accepted or paid for, the contract is enforceable without a writing. However, the contract is not enforceable beyond the quantity of goods accepted or paid for. Thus, if only some of the goods called for in the oral contract are accepted or paid for, the contract is only partially enforceable. A contract for specially manufactured goods, i.e., goods that are to be specially manufactured for the buyer and are not suitable for sale to others by the seller in the ordinary course of his business, can sometimes be enforceable without a writing, but only under circumstances where the seller has reasonably indicated that the goods are for the buyer and made a substantial beginning in their manufacture or committed for their purchase before notice of a repudiation was received. There is no exception to the Statute of Frauds for unique goods. There is no exception to the Statute of Frauds for contracts in which the goods have been shipped. There is an exception once the goods have been received and accepted or paid for.

Under the UCC, in a single delivery contract, a seller may always cure a shipment that the buyer has rejected because of defects by delivering conforming goods within: response - correct A Ten days after the notice of rejection is sent B Ten days after the notice of rejection is received Correct C The time originally provided for performance D A reasonable time beyond the original contract time

If the buyer has rejected goods because of defects, the seller may, within the time originally provided for performance, "cure" by giving reasonable notice of her intention to do so and making a new tender of conforming goods which the buyer must then accept. Ordinarily, the seller has no right to cure beyond the original contract time. However, in cases where the buyer rejects a tender of nonconforming goods that the seller reasonably believed would be acceptable "with or without money allowance," the seller, on reasonable notification to the buyer, has a further reasonable time beyond the original contract time within which to make a conforming tender. A seller will probably be found to have had reasonable cause to believe that the tender would be acceptable if the seller can show that trade practices or prior dealings with the buyer led the seller to believe that the goods would be acceptable, or the seller could not have known of the defect despite proper business conduct. There is no set time period, such as 10 days after the notice of rejection is sent or received, in which the seller always has a right to cure. As stated above, the length of time to cure depends on the specifics of the contract and the circumstances surrounding the defective delivery.

A landowner and a purchaser orally agreed that the landowner would convey 20 acres of his 160-acre farm to the purchaser. At the time of their agreement, the landowner wrote on the back of an envelope, "I hereby promise to convey the northern 20 acres of my farm to [the purchaser] for $10,000." One month later, the purchaser tendered $10,000 to the landowner, but the landowner refused to convey the 20 acres. If the purchaser sues the landowner to convey the land and the landowner prevails, what will be the most likely reason? response - correct Correct AThe writing was not signed by the landowner. BThe writing was not signed by the purchaser. CThe writing did not describe the property with specificity. DThe writing was on the back of an envelope.

If the landowner prevails, it will be because the writing was not signed by the landowner. Under the Statute of Frauds, to be enforceable a contract for the sale of land must be evidenced by a writing signed by the party sought to be charged. Here, the landowner is the party that the purchaser is seeking to charge, so his signature is required on the writing. (B) is wrong because the purchaser's signature is not required to bind the landowner. (C) is wrong because the contract need only reasonably describe the subject matter; great specificity, such as a legal description, is not required. Nothing in the facts suggests that "the northern 20 acres of my farm" is not an adequate description, which makes (C) a less certain reason than (A) why the landowner would prevail. (D) is wrong because it does not matter on what substance the writing is made.

The owner of a one-acre parcel of land with a small house on it rented the property to a professor of a nearby college at a monthly rental of $500. Several years later, after the professor got tenure, the parties orally agreed that the professor would purchase the property from the owner for the sum of $60,000, payable at the rate of $500 a month for 10 years. They agreed that the owner would give the professor a deed to the property after five years had passed and $30,000 had been paid toward the purchase price, and that the professor would execute a note secured by a mortgage for the balance. The professor continued in possession of the property and made all monthly payments in a timely fashion. When he had paid $30,000, he tendered a proper note and mortgage to the property owner and demanded that she deliver the deed as agreed. The owner refused because valuable minerals had been discovered on adjacent parcels in recent months, causing the value of this parcel of land to increase to 10 times its former value. The professor brought suit against the property owner for specific performance. If the court rules in favor of the property owner, what is the likely reason? response - correct AThe transaction had not proceeded far enough to amount to an estoppel against enforcement of the Statute of Frauds. BThe purchase price, given the present value of the land, made the contract unconscionable, providing the property owner with a valid defense to enforcement. COral agreements are generally revocable unless expressly made irrevocable. Correct DThe professor's payments are as consistent with there being a landlord-tenant relationship between them as with there being an oral contract.

If the property owner wins, it will be because the payments by the professor may be based on a valid landlord-tenant relationship. A promise creating an interest in land must be in writing to be enforceable. This includes not only agreements for the sale of real property or an interest therein, but also leases for more than one year. However, under the part performance doctrine, conduct that unequivocally indicates that the parties have contracted for the sale of land will take the contract out of the Statute of Frauds. Here, the parties had originally created a landlord-tenant relationship, and the lease would be enforceable even without a writing as a month-to-month tenancy. The continuation of the monthly payments can as readily be explained by a continuation of the lease relationship as by an oral agreement for an installment land sale contract. Thus, because the conduct does not unequivocally indicate a contract for the sale of land, the Statute of Frauds requirements will not be excepted. (A) is wrong because while part performance may create an estoppel, the professor will have a hard time proving it because the parties' conduct is consistent with a lease relationship as well. (B) is wrong because unconscionability is measured at the time the contract is formed, and there is nothing in the facts to indicate that the price was not fair at that time. Moreover, the property owner was not in a weaker bargaining position vis-??-vis the professor that would have forced her to accept an unfair price for the property; the parties were of roughly equal bargaining position and, as discussed above, the price was not unfair when the deal was struck. (C) is wrong because it states an incorrect position of the law; other than Statute of Frauds requirements, oral agreements are no more revocable than written agreements.

The right to specific performance in a land sale contract is cut off if the subject matter of the contract has already been sold to another who purchased for value and in good faith. This is known as the equitable defense of: response - correct A Laches Correct B Sale to a bona fide purchaser C Replevy D Unclean Hands

If the subject matter of a goods or land contract has already been sold to another who purchased for value and in good faith, the right to specific performance is cut off. This is known as the equitable defense of sale to a bona fide purchaser. The equitable defense of laches arises when a party delays in bringing an equitable action and the delay prejudices the defendant. Note that mere delay itself is not a ground for this defense. The unclean hands defense arises when the party seeking specific performance is guilty of some wrongdoing in the transaction being sued upon. Note that the wrongdoing must be related to the transaction being sued upon; it is not sufficient that the plaintiff has defrauded other persons in similar transactions. Replevy is a nonmonetary remedy found in Article 2 of the UCC. If a buyer has made at least part payment of the purchase price of goods that have been identified under a contract and the seller has not delivered the goods, the buyer may replevy (or recover) the goods from the seller if the seller becomes insolvent within 10 days after receiving the buyer's first payment or the goods were purchased for personal, family, or household purposes. In either case, the buyer must tender any unpaid portion of the purchase price to the seller.

Under the UCC, in the case of a single delivery contract, if the buyer finds that the goods are defective at the time of delivery: response - correct A The buyer must accept the defective goods, but must allow the seller to cure the defects within the original time for performance of the contract Correct B The buyer can reject the goods, but the seller then has a right to cure within the original time for performance of the contract C The buyer must accept the defective goods, but may recover from the seller whatever damages she has suffered as a result of the defects D The buyer can reject the goods outright, thus placing the seller in immediate breach with no right to cure

In a single delivery contract, a buyer can reject defective goods, but the UCC grants the seller the right to cure the defective shipment within the original time for performance of the contract. A seller cures by giving reasonable notice of his intention to do so and making a new tender of conforming goods within the time originally provided for performance in the contract. A buyer is not required to accept the defective goods. But if the seller properly cures the defect, the buyer must accept the cure or will be in breach of contract.

In a single delivery contract, when a buyer rejects goods due to defects, the seller may cure within the time originally provided for performance in the contract: response - correct Aif the seller reasonably believed the goods would be acceptable with or without money allowance. Correct Bby giving reasonable notice to the buyer and making a new tender of conforming goods, which the buyer must then accept. Cif the nonconforming goods did not substantially impair the value of the entire contract. Dby giving reasonable notice to the buyer and making a new tender of conforming goods, which the buyer then has the option to accept or reject.

In a single delivery contract, if the buyer has rejected goods because of defects, the seller may, within the time originally provided for performance, cure by giving reasonable notice of her intention to do so and making a new tender of conforming goods, which the buyer must then accept. If the new tender of conforming goods is made within the time originally provided for performance, the buyer does not have the option to accept or reject the goods. The buyer must accept the conforming goods. Whether the seller reasonably believed the goods would be acceptable with or without money allowance is the standard used in determining whether the seller should be granted time to cure beyond the original time provided for performance. It has no bearing on the seller's right to cure within the original time provided. Whether the nonconforming goods substantially impair the value of the entire contract is the standard for canceling an installment contract. It does not apply to a seller's right to cure.

Certain agreements, by statute, must be evidenced by a writing signed by the parties sought to be bound. This is commonly known as: response - correct Correct A The Statute of Frauds B The Confirmatory Memo Rule C The Perfect Tender Rule D The Parol Evidence Rule

In most instances, an oral contract is valid. However, certain agreements must be evidenced by a writing signed by the parties sought to be bound as set forth in the Statute of Frauds. These agreements include things such as suretyship promises, promises creating an interest in land, promises that cannot be performed within one year, and contracts for the sale of goods priced at $500 or more. The Confirmatory Memo Rule is a special UCC rule used for confirming oral agreements. In contracts between merchants, if one party, within a reasonable time after an oral agreement has been made, sends to the other party a written confirmation of the understanding that is sufficient under the Statute of Frauds to bind the sender, it will also bind the recipient if: (i) he has reason to know of the confirmation's contents; and (ii) he does not object to it in writing within 10 days of receipt. Under the Perfect Tender Rule, if goods or their delivery fail to conform to the contract in any way, the buyer may reject all, accept all, or accept any commercial units and reject the rest. The Parol Evidence Rule is a rule used in interpreting and enforcing a written contract. Under that rule, when the parties to a contract enter into an agreement in writing, with the intent that the agreement embodies the final expression of their bargain, any other expressions, written or oral, made prior to the writing, as well as any other oral expressions made contemporaneous with the writing, are inadmissible to vary the terms of the writing.

On July 1, a cattle rancher offered to sell his ranch to a dairy farmer for $150,000. The dairy farmer paid the cattle rancher $1,000 to hold the offer open for a period of 30 days. On July 10, the dairy farmer wrote to the cattle rancher, telling him that he could not pay more than $100,000 for the ranch, and that if he would not agree to accept that amount, he would not go through with the deal. The dairy farmer received no reply from the cattle rancher. On July 29, the dairy farmer mailed a letter to the cattle rancher telling him that he accepted his offer to sell the ranch and enclosed a check for $150,000. The cattle rancher received this letter on August 1. Has a contract been formed between the parties for the sale of the ranch? response - correct ANo, because the dairy farmer's letter of July 10 terminated the cattle rancher's offer. BNo, because the cattle rancher did not accept the dairy farmer's counteroffer of $100,000. Correct CNo, because the cattle rancher did not receive the dairy farmer's acceptance within 30 days. DYes, because the dairy farmer dispatched his acceptance of the cattle rancher's offer prior to the expiration of 30 days.

No contract was formed because the cattle rancher did not receive the dairy farmer's acceptance within 30 days. Under the mailbox rule, acceptance by mail or similar means creates a contract at the moment of dispatch. However, the mailbox rule does not apply to option contracts. An acceptance under an option contract is effective only upon receipt. [Restatement (Second) of Contracts §63] Here, an option contract existed because the dairy farmer paid the cattle rancher $1,000 to hold the offer open for 30 days. The dairy farmer mailed his acceptance within 30 days but it was not received by the cattle rancher within the 30-day period, so the acceptance was not effective. The option specified the period of time during which the offer would remain open, after which the offer terminated. Thus, (C) is correct, and (D) is wrong. (A) and (B) are wrong because an option contract is irrevocable for the time period stated. Thus, not even the dairy farmer himself could revoke the offer within the 30-day period.

Under Article 2, when an offeree proposes additional or different terms as part of an otherwise valid acceptance, the acceptance __________. response - correct A Fails under the mirror image rule B Fails under the battle of the forms Correct C Is effective, unless the acceptance is expressly made conditional on assent to the additional or different terms D Is deemed a rejection and counteroffer

The Article 2 battle of the forms provision provides that the proposal of additional or different terms by the offeree in a definite and timely acceptance is effective as an acceptance, unless the acceptance is expressly made conditional on assent to the additional or different terms. Whether the additional or different terms become part of the contract depends on whether or not both parties are merchants. Article 2 has abandoned the mirror image rule, which insists on an absolute and unequivocal acceptance of each and every term of the offer. Under that rule, any different or additional terms in the acceptance make the response a rejection and counteroffer.

A nonbreaching party may not seek specific performance: response - correct A when the contract is for the sale of land Correct B when a service contract is involved C when the legal remedy is inadequate D when the subject matter of the contract is rare

Specific performance is not available for breach of a service contract. One reason is the difficulty in supervising the performance, but the primary reason is that courts feel it is tantamount to involuntary servitude. A nonbreaching party may seek specific performance only when the legal remedy is inadequate, such as when the subject matter of the contract is rare or unique. Specific performance is always available for land sale contracts because all land is considered to be unique.

A writing is not required to enforce a contract that would otherwise be covered by the Statute of Frauds if: response - correct Correct A The party against whom enforcement is sought admits to the existence of the contract in court B The party seeking to enforce the contract offers evidence of prior dealings between the parties C Both parties are nonmerchants D Both parties are merchants

The Statute of Frauds requires that certain contracts be evidenced by a writing signed by the parties sought to be bound. However, if the party against whom enforcement is sought admits in pleadings, testimony, or otherwise in court that the contract was made, the contract is enforceable without a writing (but in such a case the contract is not enforced beyond the quantity of goods admitted). Prior dealings alone are not enough to remove a contract from the requirements of the Statute of Frauds. Just because the parties had an agreement in the past does not mean there is sufficient proof of a current agreement. In some such cases, estoppel could be applied where it would be inequitable to allow the Statute of Frauds to defeat a meritorious claim, but for that to occur the party seeking to enforce the agreement would need to show that enforcement is necessary to prevent injustice. An example of this would be if the other party falsely and intentionally told the plaintiff the contract is not within the Statute of Frauds or said he would reduce the agreement to a writing but failed to do so. The Statute of Frauds applies to both merchants and nonmerchants. There is no general exception to the Statute of Frauds just because both parties are merchants. Note, however, that under the merchant's confirmatory memo rule, in contracts between merchants, if one party, within a reasonable time after an oral agreement has been made, sends to the other party a written confirmation of the understanding that is sufficient under the Statute of Frauds to bind the sender, it will also bind the recipient if the recipient has reason to know of the confirmation's contents and does not object to it in writing within 10 days of receipt.

The UCC gives a seller the right to cure a defective shipment within a reasonable time beyond the original time for performance in the contract if: response - correct A The buyer would suffer no damages by allowing the seller to cure Correct B Prior dealings with the buyer led the seller to reasonably believe that the defective shipment would be acceptable C The buyer agrees to extend the terms of the contract to allow for late delivery without additional consideration D The seller notifies the buyer of his desire to cure before the time for performance in the original contract

The UCC provides that in cases where a buyer rejects a tender of nonconforming goods that the seller reasonably believed would be acceptable "with or without money allowance," the seller, upon a reasonable notification to the buyer, has a further reasonable time beyond the original contract time within which to make a conforming tender. A seller will probably be found to have had reasonable cause to believe that the tender would be acceptable if the seller can show that trade practices or prior dealings with the buyer led the seller to believe that the goods would be acceptable, or the seller could not have known of the defect despite proper business conduct. Although notification of the desire to cure is required before the seller can cure the defective delivery, it is not enough that the seller notifies the buyer of his desire to cure before the time for performance in the original contract. To extend the right to cure beyond the original time of performance, the seller must show that he mistakenly, but reasonably, believed that the nonconforming goods would be acceptable to the buyer. The seller's right to cure under the UCC is not determined by whether the buyer would suffer damages by allowing the seller to cure; and the buyer need not agree to extend the terms of the contract to allow for late delivery without additional consideration for the seller to exercise the right to cure under the UCC.

A steelmaker purchased a tube rolling machine from a manufacturer of heavy machinery. The machine was sold unassembled for a price of $150,000, with $25,000 payable on delivery and the balance ($125,000) to be paid in 10 monthly installments of $12,500 each. After the machine parts were delivered, the steelmaker contacted an assembly company that specialized in assembly and installation of large and complex manufacturing machinery, and told the company that the machinery had to be up and running within 45 days, or the steelmaker would be in breach of a major contract that it relied on for much of its current revenue. The company agreed, in a written contract with the steelmaker, to assemble and install the tube rolling machine within 45 days at a price of $15,000. Two weeks later, the manufacturer that sold the tube rolling machine to the steelmaker learned that the assembly company was planning to stop work, due to a strike by its labor union. The manufacturer orally offered the assembly company a $3,500 bonus if it would agree to finish the job for the steelmaker. The company accepted the manufacturer's promise and completed the assembly and installation of the tube rolling machine with supervisory personnel within the 45-day time limit set in the agreement between the company and the steelmaker. However, the manufacturer refused to pay the assembly company the $3,500 bonus, so the company sued the manufacturer. Which of the following would be the assembly company's strongest argument to prevail? response - correct Correct AThe assembly company owed the manufacturer no preexisting duty to complete the job for the steelmaker, and such completion was sufficient bargained-for consideration for the manufacturer's promise to pay the additional $3,500. BBecause the $3,500 payment was characterized as a "bonus," no further consideration was required and the manufacturer is bound to its promise. CThe assembly company would not have completed the job for the steelmaker within the time limit except in reliance on the manufacturer's promise to pay the additional $3,500. DBy completing the job for the steelmaker, the assembly company conferred a benefit on the manufacturer worth at least $3,500, because such performance assured the steelmaker's ability to pay the manufacturer the balance on the installment purchase agreement for the tube rolling machine.

The assembly company's best argument is that it owed the manufacturer no preexisting duty to complete the job, and such completion was sufficient bargained-for consideration. Generally, a promise is unenforceable unless it is supported by consideration; thus, for the manufacturer's promise to be enforceable, there must be consideration supporting it. Consideration is defined as a bargained-for exchange of something of legal value. Most courts hold that the thing exchanged will have legal value if it causes the promisee to incur a detriment. A minority of courts hold that a benefit to the promisor is also sufficient. Thus, the company's best argument would be one that includes the idea that it incurred a bargained-for detriment, and this is reflected by (A). The problem with (A) is the preexisting legal duty rule. Traditionally, courts have held that performance of an existing legal duty is not sufficient consideration. However, the rule is riddled with exceptions, and one exception recognized in most jurisdictions applies when, as here, the preexisting duty is owed to someone other than the promisor. Thus, (A) is the best argument because it provides for a full contract recovery. (D) is wrong because it merely reflects the fact that the manufacturer received a benefit. As indicated above, it is the presence of consideration-defined as a bargained-for exchange of something of legal value-that permits the contract to be fully enforced. (A) is a better answer than (D) because it more clearly reflects the basis for finding consideration here. (B) is wrong because merely identifying a promise to pay as a "bonus" does not obviate the need for consideration. For a promise to be enforceable, there must be consideration. (C) is wrong because mere reliance on a promise is not enough to make a contract enforceable. For reliance to provide a substitute for consideration, under the doctrine of promissory estoppel, the promisor must reasonably expect that its promise will induce reliance, and such reliance must reasonably be induced. However, the promise will be enforceable only to the extent necessary to prevent injustice. Here, because the company had a duty to complete the work even without the manufacturer's promise, there is no indication that justice would require payment of the $3,500; there is nothing in the facts to show the company incurred more costs, etc. Thus, the recovery to the company under a promissory estoppel theory would undoubtedly be less than the contract recovery possible under (A).

Under Article 2, when an offeree proposes additional or different terms during acceptance, the court will apply __________ to determine whether the additional or different terms become part of the contract. response - correct A gap fillers Correct B the battle of the forms provision C the mirror image rule D the mailbox rule

The battle of the forms provision of Article 2 lists specific rules for determining what terms are included in a contract when the terms of acceptance do not match the terms of the offer. Article 2 has abandoned the mirror image rule, which requires an absolute and unequivocal acceptance of each and every term of the offer. Gap fillers are used when certain terms are not included in the contract; it does not apply to additional or different terms in the acceptance. The mailbox rule is applied to determine the timing of acceptance of a contract.

On April 1, a graduate student who owned an antique dictionary agreed to sell it to a buyer for $1,500. The written contract between the seller and the buyer provided that the dictionary would not be delivered to the buyer until April 20. Late on April 15, a fire swept through the seller's apartment building, through no fault of the seller, and the dictionary was destroyed. Fortunately for the seller, he had insurance that covered all of his damages, including compensation for the destroyed dictionary. On April 20, the seller told the buyer of the fire, but still demanded payment, claiming that the buyer was the equitable owner of the dictionary when it was destroyed, and told her that she could have obtained insurance on the dictionary had she wanted to, because she had an insurable interest in the dictionary as soon as the contract was made. The buyer refused to pay. The seller brings an action against the buyer for the $1,500. Who will prevail? response - correct AThe buyer, because the seller was fully compensated for his dictionary and making the buyer pay would therefore result in unjust enrichment. Correct BThe buyer, because destruction of the dictionary avoids the contract and discharges her duty to pay. CThe seller, because when he contracted with the buyer, the risk of loss passed to her. DThe seller, because of the doctrine of equitable conversion.

The buyer will prevail because complete destruction of the dictionary results in avoidance of the contract and discharge of her duty to pay, since the seller still had the risk of loss. Because the contract here is for the sale of goods, it is governed by the Uniform Commercial Code ("UCC"). Under the UCC, if a contract requires for its performance particular goods identified when the contract is made, and, before risk of loss passes to the buyer, the goods are destroyed without the fault of either party, the contract is avoided. [UCC §2-613] All of the elements of section 2-613 are present here. The contract required the seller's particular dictionary, which was identified at the time the contract was made. The risk of loss had not yet passed to the buyer because, in a sale by a nonmerchant such as the seller, risk of loss does not pass to the buyer until tender [UCC §2-509], and the seller never tendered the dictionary here (there was no actual tender and delivery was not due until April 20). Finally, the goods were destroyed by a fire and without the fault of either party. Thus, the contract is avoided. (The same conclusion would result under the common law doctrine of impossibility-all executory duties are discharged when the subsequent destruction of the subject matter of a contract renders performance impossible.) (A) is wrong because the UCC contains no such rule. The only UCC remedy that depends on an injured party's insurance involves the risk of loss after the buyer's revocation of acceptance or wrongful repudiation under section 2-510. Here, the buyer does not have to pay because the destruction of the dictionary discharged her duty to do so. (C) is wrong because, as explained above, the risk of loss had not yet passed to the buyer. (D) is wrong because the UCC does not follow the doctrine of equitable conversion; rather, the Code contains very specific risk of loss rules, as detailed above.

A recent nursing school graduate mailed a letter to a classmate on July 1 telling her that she was moving to take a nursing position in another city and asking her whether she wanted "the stuff in my house" for $2,500. The classmate received the letter on July 2, and on July 3 she sent the newly minted nurse a letter accepting the offer. The next day the classmate changed her mind, called the nurse, and told her to forget the deal. Later that same day, the nurse received the letter that her classmate had sent on July 3. Is there a contract between the nurse and her classmate? response - correct AYes, because the contract is for the sale of goods for more than $500 and the classmate's attempted rejection is oral. Correct BYes, because the classmate's letter of acceptance was effective when she mailed it. CNo, because the classmate's rejection was communicated to the nurse before her letter of acceptance was received. DNo, because the description of the subject matter as "the stuff in my house" is not sufficiently definite and certain.

The classmate accepted the nurse's offer when she mailed the letter on July 3; thus, a contract was formed. Under the mailbox rule, acceptance of an offer by mail creates a contract at the moment the acceptance is posted, properly stamped, and addressed. If the offeree sends both an acceptance and a rejection, whether the mailbox rule will apply depends on which the offeree sent first, the acceptance or the rejection. If the offeree first sends an acceptance and later sends her rejection, the mailbox rule does apply. Thus, even if the rejection arrives first, the acceptance is effective upon mailing (and so a contract is formed) unless the offeror changes his position in reliance on the rejection. Here, the classmate first sent an acceptance, then called with her rejection. The mailbox rule applies, and because there is nothing in the facts to show that the nurse relied on the rejection, a contract was formed. (A) is wrong because it implies that a rejection must be in writing. There is no such requirement. Also, the rejection (absent detrimental reliance) has no effect on the contract because the offer had already been accepted and the contract formed. (C) is wrong because, as stated above, under the mailbox rule the fact that the rejection was received before the acceptance is irrelevant (unless there has been detrimental reliance on the rejection, which was not the case here). The contract was formed when the classmate sent her acceptance. (D) is wrong because the description, although somewhat ambiguous, can be made reasonably certain by evidence of the subjective understanding of the parties and extrinsic evidence of what was in the house, which a court will consider to clarify an ambiguous term.

An art collector was interested in buying a painting from his neighbor. The neighbor told the collector that he could have the painting for $30,000. The collector wanted to think the purchase over. Therefore, the two agreed in writing that the neighbor would keep the offer open for 30 days in exchange for $500, which the collector paid. The terms of the written agreement provided that the offer would expire at 11:59 p.m. on September 30 if the collector failed to accept by that time. On September 20, the collector telephoned his neighbor and told him, "The more I think about it, the less I think that I want your painting." The neighbor responded, "That's your decision to make." On September 26, one of the neighbor's friends was visiting him, saw the painting, and offered his friend (the neighbor) $35,000 for it. On September 27, the neighbor mailed a $50 check to the collector with a letter stating that he was terminating his offer to the collector regarding the painting and refunding 10% of the money that the collector paid him to keep the offer open. He mailed the letter at 11:59 p.m. on September 27. The collector received the letter at 11:30 a.m. on September 29. On September 28, at 9:30 a.m., the collector mailed a letter to his neighbor stating that he had decided to purchase the painting and a certified check in the amount of $30,000 was enclosed. Two hours later, the neighbor sold the painting to his friend for $35,000. The neighbor received the collector's letter on October 1 and immediately mailed the check back to the collector. Can the collector maintain a successful legal action against his neighbor? response - correct AYes, because the neighbor sold the painting after the collector's effective acceptance, and before the neighbor's revocation became effective. BYes, because in his revocation the neighbor did not refund the full $500 to the collector. CNo, because the neighbor effectively revoked his offer before the collector accepted. Correct DNo, because the collector's power to accept lapsed before he effectively accepted.

The collector's power to accept lapsed because the option contract specified that the offer would expire at 11:59 p.m. on September 30. Hence, the power had to be exercised prior to that time and it was not. The mailbox rule does not apply to the exercise of options. In such cases, acceptance is effective when received by the offeror, here on October 1. Thus, (D) is correct. (A) is wrong because, for the reasons discussed above, the collector did not effectively accept before his option expired. (C) is wrong for two reasons: (i) a revocation is not effective until received; and (ii) because the contract is an option, the offeror's power to terminate the offer through revocation is limited. Even if the revocation had arrived earlier, the neighbor lacked the power to revoke. (B) is irrelevant. Returning the consideration, in and of itself, would not give the offeror the power to revoke in an option situation.

On January 30, a company that designs and builds generators to standard industrial specifications received a telephone call from a buyer who ordered two generators at a price of $25,000 each. The parties agreed that delivery of the first generator would be on March 15, and the second on April 30. Payment was to be made no more than 30 days after delivery. On March 12, the company delivered the first generator, which the buyer accepted. On April 22, the company completed the second generator but had not yet notified the buyer. On April 23, the buyer, having made no payment to the company, canceled the agreement. The company brings an action against the buyer for breach of contract. How much should the company recover in damages? response - correct ANothing. Correct B$25,000 only. CDamages for total breach of contract for the sale of two generators, because the buyer accepted part performance. DDamages for total breach of contract for the sale of two generators, because the goods were made for the buyer.

The company should recover $25,000 only. Contracts for goods for $500 or more must be evidenced by a writing to be enforceable. There are three exceptions to this rule: specially manufactured goods unsuitable for resale in the seller's regular course of business, contracts admitted in court, and contracts partially accepted (enforceable to the extent of the acceptance). Here, the contract was for $50,000 and was oral. Thus, it will be enforceable only if one of the exceptions applies. The buyer's acceptance of the first generator constitutes part acceptance that will make the buyer liable to the extent of the acceptance: $25,000. Therefore, (B) is correct and (A) is incorrect. (C) is incorrect because partial acceptance renders the buyer liable only for the part accepted, not the entire contract. (D) is incorrect because, while the goods were made for the buyer, they were suitable for resale in the company's business, because they were built to standard industry specifications.

A homeowner and a contractor duly executed a contract providing that the contractor was to construct a residence on a specified lot. No date was included in the contract for completion of the home. After the contractor completed 5% of the residence, a tornado demolished the construction but left the lot undamaged. Which of the following states the probable legal consequences of the tornado damage? response - correct AThe contract is void because the subject matter of the contract was destroyed through no fault of the parties. BThe contractor's duty of performance is discharged because of impossibility. CThe contractor remains obligated to construct the residence, but he is entitled to a quantum meruit recovery for the work done prior to the tornado. Correct DThe contractor remains obligated to perform under the original contract without any compensation for the work done prior to the tornado.

The contractor remains bound under the original contract, and he is not entitled to compensation for the work that was destroyed. The general rule is that a contractor is responsible for destruction of the premises under construction prior to completion. Once the residence is completed, risk of loss shifts to the owner. (A) is wrong because the subject matter was not destroyed. Note that even if the subject matter were destroyed, it would not void the contract; it would merely discharge the contractor's duties under the contract. (B) is wrong because performance is not impossible; the contractor can rebuild the residence.

A property owner decided that she would turn the garage on her property into an exercise area, including a modern sauna and spa. She entered into a written agreement with a contractor who agreed to do the job personally for $12,500, which included all requisite plumbing, electrical, and carpentry work. The contractor was to begin work by May 14. On May 15, he had not yet appeared to start the job. The property owner telephoned the contractor, who told her that he was hired for a big job that was going to pay him "a lot more money than that marginal project of yours, so I'm not going to work on your garage." Over a period of several months, the property owner made many calls to local contractors, but none of them would agree to do the job for the price agreed upon by the original contractor. On June 3 of the following year, the property owner filed suit for specific performance against the original contractor. Which of the following represents the contractor's best argument in his defense against the property owner's suit? response - correct ASpecific performance is an equitable remedy, and because the property owner waited for over a year to sue, the equitable defense of laches will apply. Correct BSpecific performance is inappropriate, because a contract for services is involved. CSpecific performance is inappropriate, because nominal legal damages are available to the property owner. DSpecific performance is inappropriate, because the property owner's failure to obtain another contractor for the job is an indication that $12,500 was an unfair price.

The contractor's best argument is that a contract to provide services is not specifically enforceable. Thus, the property owner cannot obtain specific performance of the contractor's agreement to perform personal plumbing, electrical, and carpentry work for her. One of the prerequisites to obtaining specific performance is that a plaintiff must show that the legal remedy is inadequate. Where the plaintiff has contracted for something rare or unique, money damages are inadequate compensation for loss of the bargain. Generally, the services to be performed under a contract for services are not unique and money damages can remedy a breach. Thus, specific performance is not available in such cases. In addition, even in the case of unique services, a court will not order a defendant to work for the plaintiff, in part because of the difficulty of enforcement and because such an order is tantamount to unconstitutional involuntary servitude. Another requirement for specific performance is that enforcement must be feasible. Enforcing a services contract generally would create complicated and time-consuming supervision problems that courts are reluctant to undertake. In this case, the contractor agreed to personally perform for the property owner plumbing, electrical, and carpentry work. Thus, this was a contract to provide services. However, the services to be performed by the contractor were not unique or capable of being performed solely by him. The property owner could obtain adequate compensation by receiving the amount, above her contract price with the contractor, that it will cost to have someone else perform the required work (plus reasonable compensation for the delay in performance). Thus, specific performance is inappropriate here. (A) is incorrect because circumstances that would permit the defense of laches do not appear to be present. Laches is available as an equitable defense if the plaintiff has unreasonably delayed in bringing the action and the delay is prejudicial to the defendant. There is no automatic invocation of laches by a delay of one year before suit is filed. Here, there is no showing that the property owner's delay in filing suit was unreasonable, given that she spent several months trying to find another contractor, nor that the contractor has been prejudiced by the delay. Therefore, laches will not provide the contractor with a strong defense. (C) is incorrect because nominal damages would not be an adequate legal remedy. Nominal damages are appropriate where there is a breach, but no actual loss. Here, nominal damages would fail to compensate the property owner for the amount she will have to pay above the price agreed to by the contractor. Thus, the availability of nominal damages would not, by itself, render specific performance an inappropriate remedy. Regarding (D), a court of equity may inquire into the relative values of agreed-upon consideration and deny an equitable remedy if it finds a contract to be unconscionable. Nevertheless, the mere fact that the property owner could not find another contractor to do the job for the price agreed upon by the contractor does not establish that the contract was unconscionable. Thus, the contractor will not be successful in his contention that specific performance should be denied on the basis that the price for which he agreed to do the work was too low.

A wholesale seller of widgets telephoned a retail seller of widgets and told him that he had 5,000 pounds of widgets ready for delivery at $5,000. The retailer agreed to purchase the widgets, but stated that he wanted the wholesaler to deliver 2,000 pounds now and 3,000 pounds next month. There were no further communications between the parties. Assuming that the retailer's request is not a material change of terms, what is the most likely result of the conversation between the wholesaler and the retailer? response - correct Correct AA contract was formed to deliver 2,000 pounds now and 3,000 pounds next month. BA contract was formed to deliver 5,000 pounds now. CNo contract was formed, because the retailer's response was merely a counteroffer and a rejection. DNo contract was formed, unless the wholesaler notified the retailer within a reasonable time of his assent to the proposed schedule of delivery.

The conversation created a contract for 2,000 pounds of widgets now and 3,000 pounds next month. Because the contract is for the sale of goods, the UCC governs. Under the UCC, a contract is formed whenever it appears from the parties' communications that they intended to enter into a contract. Here, it is clear that the parties intended to enter into a contract, but the acceptance contained terms additional to the offer terms. When this occurs, the UCC provides for which terms govern: If the contract is between merchants, the additional terms in the acceptance are included in the contract, unless (i) the additional terms materially alter the contract, (ii) the offer expressly limits acceptance to the terms of the offer, or (iii) the offeror objects within a reasonable time. Here, both parties are merchants, and we were told to assume that that the delivery terms do not materially alter the contract. There is no indication that the offer limited acceptance to the terms of the offer or that the wholesaler objected to the terms; thus, there is a contract containing the additional terms. (B) would be correct if one of the parties were not a merchant, because under the UCC, when an acceptance proposes additional terms, a contract would be formed under the terms of the offer unless both parties are merchants. (C) would be correct if the UCC did not apply, because under the common law, an acceptance must mirror the offer (the "mirror image" rule); if new terms are added in the acceptance, it is treated as a counteroffer. (D) is incorrect because, under the UCC, no notice was necessary to form the contract. Notice would be required, however, if the wholesaler did not want to be bound by the additional terms. Note that the fact that this was an oral conversation does not prevent formation of the contract. To be enforceable, any contract for the sale of goods priced at $500 or more must be evidenced by a writing signed by the party to be charged. This affects enforceablility, not formation. For this contract to be enforceable, some form of signed writing (e.g., a merchant's confirmatory memo) would be necessary.

The owner of a summer cottage contracted to put new vinyl siding on the cottage for $10,500. Two weeks before the work was to start, however, the contractor called to say that there was a clerical error in the bid and that he could not do the work for less than $12,000 or he would lose money. The cottage owner agreed to pay the additional $1,500 but told the contractor that he was being unfair. After the work was completed, the cottage owner handed the contractor a check for $10,500, telling the contractor that that was all he would pay him because he had no right to raise the price. If the contractor sues the cottage owner for the additional $1,500, who will prevail? response - correct Correct AThe cottage owner, because the contractor was already under a preexisting legal duty to replace the siding on the cottage for $10,500. BThe cottage owner, because the promise to pay the additional money was not in writing. CThe contractor, because he relied on the cottage owner's promise to pay the additional money to his detriment. DThe contractor, because there was a valid modification of the parties' original contract.

The cottage owner will prevail, because the contractor was already under a preexisting legal duty to replace the siding on the cottage for $10,500. Under the preexisting legal duty rule, the promise to perform or the performance of an existing legal duty will not be sufficient consideration. If the parties agree to modify their contract, consideration is usually found to exist where the obligations of both parties are varied. However, absent unanticipated circumstances, a modification solely for the benefit of one of the parties is generally unenforceable in contracts not governed by the UCC. Here, the contractor was already under a binding contract to replace the siding on the cottage for $10,500. The contract is primarily for services, and thus not governed by the UCC. Moreover, the obligations of both parties under the modified agreement are not varied; the modification (paying the contractor an additional $1,500) would benefit only the contractor. The contractor's performance of a duty that he was already obligated to perform does not constitute sufficient consideration to support the modification. Therefore, the cottage owner is obligated to pay only the originally agreed-upon $10,500. Thus, (A) is correct and (D) is incorrect. (B) is incorrect because contracts for services do not come within the Statute of Frauds unless by their terms they cannot be performed within one year of their making, and performance of this contract could be completed in less than a year. (C) is incorrect because, in replacing the siding on the cottage, the contractor was simply fulfilling his contractual obligation, rather than acting to his detriment in reliance on the cottage owner's promise to pay the additional money. Even where unsupported by consideration, a promise is enforceable under the promissory estoppel doctrine to the extent necessary to prevent injustice if: (i) the promisor should reasonably expect to induce action or forbearance of a definite and substantial character; and (ii) such action or forbearance is in fact induced. The contractor was legally bound to perform the work on the cottage regardless of whether the cottage owner agreed to pay the extra $1,500. Thus, the contractor did not incur a detriment in reliance on the cottage owner's promise.

A college student was interested in renting a particular apartment because of its very distinctive features, but she was also considering a number of other options. Because she wanted some time to make up her mind, she contacted the building's rental agent and asked him to reserve her right to rent the particular apartment. They made the following written agreement: "Upon payment of $200, the student shall have the right to inform the agent that she wishes to rent the apartment any time on or before July 1. If she fails to notify the agent that she wants the apartment on or before July 1, the agent shall keep the $200. However, if she does notify the agent on or before July 1 that she desires the apartment, the agent will apply the $200 to her first month's rent." The student paid the agent $200. On June 9, the agent rented the apartment to a third party who was unaware of the agent's agreement with the student. On July 1, the student told the agent that she wanted the apartment. He refused to rent it to her and returned her $200. The student sued, requesting the court to compel the agent to rent her the apartment. The court determined that the agent breached his agreement with the student. Would it be appropriate for the court to order the agent to rent the apartment to the student? response - correct AYes, because the agent should not have rented the apartment to the third party before July 2. BYes, because the student's harm cannot be remedied with money damages. CNo, because the contract was unconscionable. Correct DNo, because the student's right to specific performance was cut off by the lease to the third party.

The court should not order the rental agent to rent the apartment to the student because her right to specific performance was cut off by the lease to the third party. A court will grant specific performance of a contract in certain circumstances (e.g., if money damages would be inadequate). While money damages generally are considered inadequate when a transfer of an interest in land is involved (including a transfer of a leasehold estate), a court will not grant specific performance if the subject matter of the contract has been transferred to a bona fide purchaser for value. A third party lessee who was unaware of the agreement between the agent and the student would be considered a bona fide purchaser for value. Thus, even assuming that all of the elements for specific performance are present, the court still would not order the rental agent to lease the apartment to the student here. (A) would be correct if a bona fide purchaser were not involved. The student and the agent created an option contract, the terms of which required the agent to rent the apartment to the student if she notified the agent on or before July 1. The agent breached the contract by renting the apartment to a third party on June 9, because renting the apartment before July 2 prevented the agent from renting the apartment to the student. All land is considered unique (and the apartment here had some very distinctive features to make this point even clearer), so specific performance would generally be an appropriate remedy for breach but for the lease to the third party. (B) would also be a good choice but for the lease to the third party. The student's harm cannot be remedied with money damages (because the apartment is unique) and that generally is a ground for granting specific performance. However, as explained above, any right to specific performance will be cut off by the lease. (C) is wrong because nothing in the facts shows the contract to be unconscionable. Unconscionability prevents oppression and unfair surprise. The basic test is whether at the time the contract was formed the clauses involved were extremely one-sided. That was not the case here.

A craftsperson entered into a written agreement with an electrician to install electrical wiring in her standalone garage so that she could convert the garage into a workshop. The contract contained a clause requiring all electrical work to be completed within two days and provided that the craftsperson would pay the electrician $700 for his work. After the first day, approximately half of the job was completed. That evening, a piece of a defunct satellite reentered the Earth's atmosphere and a large chunk of it crashed directly into the craftsperson's garage, catching the garage on fire and destroying it. Which of the following best describes the obligations of the electrician and the craftsperson after the crash? response - correct ANeither the electrician nor the craftsperson is discharged from their obligations under the contract. BNeither the electrician nor the craftsperson has any further obligations. CThe craftsperson is obliged to pay the electrician the full contract price of $700. Correct DThe electrician is discharged from his obligation but is entitled to recover from the craftsperson the fair value of the work he performed prior to the destruction of the garage.

The destruction of the garage discharges the electrician's duties due to impossibility, but the electrician has a right to recover for the reasonable value of the work he performed. Contractual duties are discharged where it has become impossible to perform them. The occurrence of an unanticipated or extraordinary event may make contractual duties impossible to perform. If the nonoccurrence of the event was a basic assumption of the parties in making the contract, and neither party has assumed the risk of the event's occurrence, duties under the contract may be discharged. If there is impossibility, each party is excused from duties that are yet to be performed. If either party has partially performed prior to the existence of facts resulting in impossibility, that party has a right to recover in quasi-contract for the reasonable value of his performance. While that value is usually based on the benefit received by the defendant (unjust enrichment), it also may be measured by the detriment suffered by the plaintiff (the reasonable value of the work performed). Here, the garage that the electrician was wiring burned down after a chunk of a satellite crashed into it. That event was of such an unexpected nature that its nonoccurrence was a basic assumption of the parties, and neither party was likely to have assumed the risk of its occurrence. Thus, it has become objectively impossible for the electrician (or anyone else) to complete the job. This impossibility will discharge both the craftsperson and the electrician from performing any contractual duties still to be fulfilled. Therefore, the electrician need not finish the wiring work, and the craftsperson is not obligated to pay the entire amount of $700. However, the electrician can recover under quasi-contract. (A) is incorrect because both parties are discharged. (B) is incorrect because it fails to account for the fact that the craftsperson will have to pay the electrician for the value of the work already performed. (C) is incorrect because, as discussed above, the craftsperson is obligated to pay for the value of the electrician's services to date, not the full contract price.

A farmer contracts with a mechanic to repaint his antique tractor for display in the upcoming county fair. Which of the following would discharge the contract by impossibility? response - correct A The unexpected death of the mechanic B The unexpected death of the farmer Correct C The destruction of the tractor by a tornado D The cancellation of the county fair due to a drought

The destruction of the tractor by a tornado would discharge the mechanic's performance due to impossibility. If a contract's subject matter is destroyed without the fault of either party, contractual duties are discharged. If the tractor was destroyed by an act of nature, it is impossible for the mechanic to repaint it. Neither the death of the farmer nor the death of the mechanic would discharge the contract by impossibility. Death or physical incapacity of a person necessary to effectuate the contract serves to discharge it, but this applies in personal service contracts where the services involved are "unique." If the services are the kind that could be delegated, the contract is not discharged by the death of the person who was to perform them. Painting a tractor probably does not qualify as a unique or artistic service that could not be delegated, so the death of the mechanic would not discharge his duties under the contract. The farmer's duty to pay for the painting could easily be performed by his estate after his death, so his death also will not result in a discharge of the contract by impossibility. The cancellation of the county fair would not make fulfillment of the contract impossible. The mechanic's duty is to paint the tractor, which he can do even if the fair is cancelled, and the farmer's duty is to pay the mechanic for his painting, which again he can do even if the fair is cancelled. It could be argued that the purpose of the contract was frustrated, but the contract would not be discharged by impossibility.

A farmer who supplies several local bakeries with grains wanted to sell his rye before the growing season was over. The farmer sent the following e-mail to a local baker: "Will sell my unprocessed rye, 20 bushels maximum, best price $100 per bushel, firm for 48 hours. /s/ Farmer." Unsure how the baker would respond, and anxious to find a buyer for the rye, the farmer made the same offer to the baker's chief competitor by e-mail later that same day. The baker was delighted to receive the offer, but needed a day or so to figure out how much rye she needed. When she accepted the farmer's offer the next day, e-mailing to him an order for 20 bushels, she was aware of the farmer's offer to her competitor, and that her competitor had also e-mailed an order to the farmer for 20 bushels. Unbeknownst to the baker, the farmer has only 30 bushels of rye left in his fields. Assuming the farmer is a merchant with respect to rye, which of the following states the probable legal consequences of the correspondence between the parties? response - correct AThe farmer has a contract with the baker and her competitor for 15 bushels each. BThe farmer has a contract with the baker's competitor for 20 bushels and a contract with the baker for the remaining 10 bushels. Correct CThe farmer has a contract with the baker for 20 bushels and a contract with her competitor for 20 bushels. DThe farmer has a contract with neither the baker nor her competitor.

The farmer has two contracts, one with the baker and one with the competitor, for 20 bushels each. Because his e-mail provided a firm price for 48 hours and the farmer is a merchant, the offer was an irrevocable firm merchant's offer during the 48 hours. Under the UCC, which governs here because goods are involved, a written offer signed by a merchant giving assurances that it will stay open will be irrevocable for the time stated. The farmer qualifies as a merchant of rye (one who deals in goods of that kind sold) and his offer was written and signed and contained words of firmness ("firm for 48 hours"), so it was irrevocable for 48 hours. The baker accepted the offer within the stated time. Thus, a contract was formed between the baker and the farmer. A contract was also formed between the baker's competitor and the farmer because the competitor accepted the farmer's offer. Therefore, the farmer is obligated to both the baker and her competitor for 20 bushels. If the farmer does not have the appropriate quantity in his field, he will have to procure it from somewhere else or be in breach. (A) and (B) are incorrect because they do not reflect the terms of the contracts agreed to by the parties. If a seller is unable to fully perform because of an unforeseen circumstance (i.e., impracticability), he must allocate deliveries between customers. First, this is not an unforeseen circumstance. Second, allocating between customers does not change those contracts. It is still a breach, and the customers may cancel the contract. (D) is incorrect because, as explained above, the farmer has a contract with both the baker and her competitor.

An advertising agency specializing in aerial banners and skywriting signed a contract with a film production company that was premiering a new blockbuster film. The contract provided that the agency would advertise the film by flying over the city towing a giant streamer belonging to the film company heralding the film's catch phrase and title in large letters. This contract specified that the flight was to be conducted on the first Saturday in June at noon (the day of the local premier), and the film company was to pay the advertising agency $500 for the flight. On the designated Saturday, the advertising agency was unable to fly because of a defective fuel pump. The defective condition was entirely unforeseeable and did not occur through any negligence or fault of the agency. The film company did not pay the agency, and each of the parties has sued the other for damages. Which of the following best states the rights and liabilities of the parties? response - correct Correct AThe film company is entitled to recover damages from the advertising agency on account of the agency's failure to fly. BThe advertising agency is entitled to recover from the film company the $500 contract price, as the incapacity of the airplane was not the agency's fault. CNeither party is entitled to recover against the other, because the advertising agency's duty to fly was discharged by impossibility, and the film company's duty to pay was contingent on the agency's flight. DNeither party is entitled to recover against the other, because the film company's offer to pay $500 for the flight was in effect an offer for an act, and because the act was not performed, there was no valid acceptance.

The film company will be able to recover damages from the advertising agency because the agency's failure to fly constituted a breach of contract. The parties entered into a bilateral contract-the agency promised to fly with the streamer and the film company promised to pay for the flight. The agency breached the contract by failing to fly on the designated Saturday. Its duty to fly was not discharged by impossibility. A contractual duty to perform may be discharged by objective impossibility (i.e., no one could have performed), but subjective impossibility (defendant could not perform) is insufficient. Here, the defect in the plane constituted only subjective impossibility (if it amounted to impossibility at all) because the agency could have obtained another plane to pull the streamer. If the agency had been unable to fly the plane because of weather (e.g., a severe ice storm), its performance would have been objectively impossible, and the agency would have been discharged. However, under these facts, the film company is entitled to damages for the agency's breach. (B) is incorrect because the film company's duty to perform (pay $500) was subject to the condition precedent of the agency's performance (flying), and, as discussed above, the agency breached the contract by failing to fly. Therefore, the film company's duty to pay never arose. The fact that the engine problem was not the agency's fault does not change things. The agency's inability to perform, even if it were due to impossibility, would merely discharge the contract, and each party would be excused from performance; the film company would not have to pay the $500. (C) is incorrect because, as determined above, the agency's duty was not discharged because performance was still possible. (If there had been objective impossibility, (C) would have been the correct choice.) (D) is incorrect because it suggests that the contract was a unilateral one (the offer to pay could be accepted only by completion of performance). This interpretation is clearly contrary to the facts. Although the film company offered to pay $500 for the flight, the agency accepted that offer by signing the contract. A promise to pay was given in exchange for a promise to fly. Thus, there was a contract to which both parties were bound.

A jeweler was commissioned by a young man to design and create a set of rings (engagement and wedding) for his fiancée. The jeweler designed and created the rings in 18k gold, leaving room in the engagement ring for a large marquise-shaped diamond. The jeweler then entered into an oral agreement with a gemologist. The terms of the agreement were that the gemologist would provide the marquise-shaped diamond and the jeweler would pay the gemologist $20,000 when the jeweler received the payment from the young man. The gemologist found and cut a suitable stone and delivered it to the jeweler, who accepted it. The gemologist waited to be paid, and when he was not, he contacted the jeweler. The jeweler refused to pay him, arguing that their agreement was unenforceable and, anyway, the young man has not paid her. If the gemologist sues the jeweler for breach of contract, what is the gemologist's likely recovery? response - correct AThe fair market value of the stone, under a quasi-contract theory. BThe cost of materials and labor, under a quasi-contract theory. Correct C$20,000, the contract price. DNothing, because the young man did not pay the jeweler.

The gemologist will be able to recover the full $20,000 contract price. Under the UCC, the contract is enforceable, despite the absence of a writing, to the extent of the goods accepted, which here is the entire amount contracted for. The proper remedy is the agreed-upon price of $20,000, which the gemologist will be able to prove by parol evidence. (A) is incorrect because the recovery will be under the contract. Because the promises are enforceable under the acceptance exception to the Statute of Frauds, the quasi-contract remedy need not be applied. Note that if the contract had been unenforceable, quasi-contract would be a basis for a recovery of restitutionary damages. (B) is incorrect because it also is a possible measure of restitutionary damages in a quasi-contract action, and as stated above, the gemologist will be able to recover under the contract here. (D) is incorrect because the court will construe the jeweler's agreement as a promise to pay at a particular time rather than an express condition for payment. If it were a condition, the jeweler would not have a duty to pay because she was not paid. However, courts prefer to construe language as a promise rather than a condition so as to reduce the obligee's risk of forfeiture. Where an agreement provides that a duty is to be performed once an event occurs, if the event is not within the control of the promisee, it is less likely that he will have assumed the risk of its nonoccurrence and therefore less likely to be a condition of the promisor's duty to perform. In doubtful situations, courts are more likely to hold that the provision is a promise rather than a condition because this supports the contract and preserves the reasonable expectations of the parties.

A hotelier opening a new inn in the Pacific Northwest sent letters to all known hotel and motel suppliers on June 1, alerting them to his need for such items as ice buckets, televisions, linen, and mattresses. The hotelier received a signed letter dated June 8 from a hotel supply company, stating that the company had 250 ice buckets left in stock and will sell them to the hotelier for $1 each. The company added that it must receive the hotelier's answer by November 1 and will hold the ice buckets for the hotelier until then. On July 1, the company sold 200 of the ice buckets to a competing hotel chain, which had recently opened a hotel on the East Coast. On July 2, the company sent the hotelier a fax stating it had only 50 ice buckets left for sale. The hotelier received the fax that day, but put it aside and never read it. On July 10, the hotelier notified the company that he was accepting the company's offer to sell 250 ice buckets. The company, upon receiving the hotelier's acceptance, shipped the remaining ice buckets. The hotelier sues the company for failing to deliver all 250 ice buckets. Will the hotelier prevail? response - correct ANo, because the hotelier is not a hotel supply merchant. BNo, because the company's offer was to remain open for more than three months. Correct CYes, because the company promised in a signed writing to hold the offer open. DYes, because the hotelier never read the company's July 2 fax.

The hotelier will prevail. Ice buckets are movable goods; therefore, Article 2 of the UCC applies. The June 8 letter from the supply company is a firm offer under UCC section 2-205. No consideration is required, because the company is a "merchant" (i.e., one who ordinarily deals in goods of the kind sold) of ice buckets. Where a time period for the offer is stated, the period of irrevocability is that period, except that the period cannot exceed three months. Here, the three-month period would end on September 8. The company's fax stating that it had only 50 ice buckets left to sell constitutes an invalid attempt at revocation, because it is within the three-month period of irrevocability. (A) is incorrect because section 2-205 does not require that the offeree of a firm offer be a merchant; it requires that the offeror be a merchant, and the company is (see above). (B) is incorrect because a firm offer that states a period longer than three months is still firm for the first three months. (D) is incorrect because the hotelier's knowledge, or lack thereof, of the "revocation" of the company's offer is irrelevant because it was invalid; the fact that the company made a firm offer prevents it from revoking the offer within the stated time, not to exceed three months.

On June 1, a jeweler placed an order for 10 classic yellow gold ring settings with a goldsmith. The jeweler's order called for delivery by July 1. On June 2, the goldsmith shipped 10 white gold ring settings that arrived at the jeweler's place of business on June 3. The jeweler immediately faxed the goldsmith rejecting the shipment because the rings were white gold instead of the classic yellow gold he had requested. The goldsmith replied by fax: "Will deliver proper ring settings before July 1." The jeweler received this fax on June 4 but did not reply to it. On June 30, the goldsmith tendered 10 classic yellow gold ring settings, but the jeweler refused to accept them. Did the jeweler properly reject the ring settings tendered on June 30? response - correct AYes, because the goldsmith's shipping the white gold settings on June 1 was a present breach of contract. BYes, because the goldsmith's shipping the white gold settings on June 1 was an anticipatory repudiation. Correct CNo, because the goldsmith cured the June 1 defective delivery by his tender of conforming goods on June 30. DNo, because a contract for the sale of goods can be modified without consideration.

The jeweler did not properly reject the ring settings. When a buyer has rejected goods because of defects, the seller may, within the time originally provided for performance, cure by giving reasonable notice of intention to do so and making a new tender of conforming goods, which the buyer must then accept. The goldsmith took all of the steps necessary to cure. Consequently, the jeweler's rejection of the ring settings tendered on June 30 was improper. It follows that (A) is incorrect. (B) is incorrect because the goldsmith's prompt sending of a fax on June 4, indicating his intention to cure the defect, does not indicate a clear unwillingness or inability to perform, as is required for anticipatory repudiation. (D) is incorrect because modification of the contract is not at issue here. By curing the defective delivery, the goldsmith is simply performing according to the terms of the contract.

On September 15, a highlighter manufacturer faxed a large office supply company offering to sell the supply company 50,000 highlighters for $25,000. The supply company faxed back the following communication: "We accept your offer. Please box 125 highlighters per case in post-consumer cardboard shipping boxes." Assuming the existence of a valid contract, what would its terms include? response - correct AOnly those terms set forth in the manufacturer's fax of September 15, because the manufacturer did not assent to any enlargement of the shipping terms. BAll terms set forth in the manufacturer's offer plus consistent additional terms proposed in the office supply company's acceptance. CAll terms set forth in the manufacturer's offer plus those in the office supply company's attempted acceptance that did not amount to a material alteration of the manufacturer's offer. Correct DAll terms set forth in the manufacturer's offer plus all those in the office supply company's purported acceptance that did not amount to a material alteration of the manufacturer's offer and to which the manufacturer did not object within a reasonable time.

The language in choice (D) properly states the UCC position regarding the terms of the contract. Under the UCC, if both parties to a contract are merchants, additional terms in an acceptance will be included in the contract unless (i) they materially alter the original contract; (ii) the offer expressly limits acceptance to the terms of the offer; or (iii) the offeror has already objected to the particular terms, or objects within a reasonable time after notice of them is received. [UCC §2-207(2)] The manufacturer and office supply company are both merchants because they regularly deal in goods. [UCC §2-104(1)] Therefore, under Article 2 the contract will include the terms of the manufacturer's offer plus those in the office supply company's purported acceptance that did not amount to a material alteration of the offer or to which the manufacturer did not object within a reasonable time. Note that the manufacturer's offer did not expressly limit acceptance of its terms. (A) is incorrect because this was a contract between two merchants. Contract formation under the UCC for contracts between merchants is governed by the rule stated above. If one of the parties were not a merchant, (A) would be correct-if one of the parties to a contract for sale of goods is not a merchant and the acceptance includes additional or different terms, such terms are considered to be mere proposals that do not become part of the contract unless the offeror accepts. [UCC §2-207(2)] However, because this is a contract between merchants, the office supply company's terms regarding shipping will be included unless they materially alter the offer, as discussed above. (B) is incorrect because it does not fully state the Code's "battle of the forms" provision. The choice fails to mention the manufacturer's power to object within a reasonable time. (C) is incorrect because, like (B), it does not note the manufacturer's power to object within a reasonable time.

On September 15, a manufacturer of office furniture received an email purchase-order form from a retailer of office furniture. The order was for 100 executive leather swivel chairs and specified a delivery date no later than November 1, at a total cost of $10,000, as quoted on the manufacturer's website. Two days later, the manufacturer emailed its own purchase-order acceptance form to the retailer, who was a new customer and had never seen the form before. The purchase-order acceptance form stated that it was an acceptance of the specified order, was signed by the manufacturer's sales manager, and contained all of the terms of the retailer's form, but it also contained an express warranty and a clause disclaiming all implied warranties such as the implied warranty of merchantability. Assuming that there were no further communications between the parties, what is the status of the relationship between the parties? response - correct AThere is an enforceable contract between the parties, the terms of which are comprised of the language in the manufacturer's form. Correct BThere is an enforceable contract between the parties, the terms of which do not include the disclaimer of implied warranties in the manufacturer's form. CThere is no enforceable contract between the parties because the manufacturer's form constituted a rejection of the retailer's offer and a counteroffer by the manufacturer. DThere is no enforceable contract between the parties because the manufacturer's form added an additional term that materially altered the terms of the retailer's offer.

The manufacturer and the retailer have a contract without the disclaimer. In contracts for the sale of goods, a definite expression of acceptance operates as an acceptance even if it states additional terms. Between merchants, additional terms proposed by the offeree in an acceptance automatically become part of the contract unless (i) they materially alter the original terms of the offer (e.g., they change a party's risk or the remedies available); (ii) the offer expressly limits acceptance to the terms of the offer; or (iii) the offeror had already objected to the additional terms or objects within a reasonable time. Here, a clause was added by the manufacturer (the offeree) providing for an express warranty and a disclaimer of all implied warranties, including the warranty of merchantability. The disclaimer materially altered the original terms of the offer. Therefore, the disclaimer would not become part of the contract. (A) is therefore incorrect. (C) is incorrect because it reflects the common law "mirror image" rule, which the UCC has rejected in sale of goods cases. (D) is incorrect because under the UCC, the inclusion of a material additional term does not prevent formation of a contract; instead, a contract is formed without the inclusion of that additional term.

A bakery owner entered into a one-year written contract with a chocolate chip manufacturer whereby the manufacturer agreed to ship to the bakery owner 5,000 pounds of premium chocolate chips on the first day of each month for $12,500 per shipment. When the bakery owner opened the first shipment of chips, he found that 60% of them had melted together. He immediately called the manufacturer and reported the breach. The manufacturer offered to send replacements, but the bakery owner refused the offer and canceled the entire contract. Will the manufacturer prevail in an action against the bakery owner seeking contract damages? response - correct ANo, because the bakery owner was entitled to a perfect tender, and the manufacturer's failure to provide a perfect tender resulted in a breach of contract. BNo, because damages are speculative. CYes, because the bakery owner's notice of the manufacturer's breach was not in writing. Correct DYes, because the manufacturer had a right to cure.

The manufacturer will prevail in an action for damages, because the manufacturer had the right to cure the defect and was prevented from doing so. In an installment contract, an installment can be rejected only if the nonconformity substantially impairs the value of that installment. Moreover, the installment contract as a whole is deemed to be breached only if the nonconformity substantially impairs the value of the entire contract. Here, the contract is an installment contract because the deliveries are to be made in separate lots at different times. The delivery of melted chocolate chips was the first of 12 deliveries that were to be made, and there is nothing in the facts to indicate that the manufacturer could not cure the problem with this delivery and with the rest of the deliveries for the year. Thus, the nonconformity did not substantially impair the value of the entire contract. As to this installment, the bakery owner must afford the chocolate chip manufacturer the right to cure, i.e., to send replacement chips as the manufacturer had offered to do, and his failure to allow the manufacturer to do so will result in the bakery owner's breach of contract. Thus, the manufacturer would prevail in an action for damages resulting from the breach. (A) is wrong because the "perfect tender" rule, which allows a buyer to reject goods if either the goods or the delivery fail to conform to the contract in any way, is not the rule for installment contracts. In installment contracts, a defective shipment cannot be rejected if the defect can be cured. (B) is wrong because damages are not speculative. Under the expectation measure, the damages are whatever it would take to put the manufacturer in the place it would have been in had the bakery owner performed under the contract. In this case, the recovery would likely include lost profits because a manufacturer of this type is typically a lost volume seller. (C) is wrong because a notification of rejection need not be in writing.

A manufacturing company was in the business of making copper tubing. A retail seller telephoned the manufacturing company's sales department and placed an order for 10,000 linear feet of copper tubing at a sale price of $2 per foot. The tubing was to be used in the production of a custom order for one of the retail seller's customers. The manufacturing company installed special equipment for the manufacture of the tubing to the retail seller's specifications and had completed a portion of the order when the retail seller again telephoned the sales department. This time, however, the retail seller canceled its order, saying it no longer had need of the tubing because its customer had been declared bankrupt and refused to pay for the order. If the manufacturing company sues for breach, will it win? response - correct Correct AYes, because the contract is fully enforceable. BYes, because the contract is enforceable to the extent of the portion of the order completed. CNo, because a contract for the sale of goods for a price of $500 or more must be in writing. DNo, because the parol evidence rule would preclude testimony about the initial telephone call.

The manufacturing company will win because the contract is fully enforceable under the UCC. Tubing is a good, so Article 2 of the UCC applies. The contract is for the sale of goods priced at $500 or more (10,000 linear feet at $2/foot), so ordinarily section 2-201 would require a writing. However, section 2-201(3) provides that a writing is not required where the contract is for "specially manufactured" goods not suitable for resale in the ordinary course of the seller's business and the seller has made a substantial beginning of their manufacture or commitments for their procurement. Because the tubing is a custom order of unique specifications and the manufacturing company has begun manufacturing it, this exception to the UCC Statute of Frauds applies. (B) is incorrect because the contract is fully enforceable. While the manufacturing company is required to mitigate damages, it is entitled to the full range of contract remedies to put it in the position it would have been in had the retail seller not breached (i.e., benefit of the bargain damages on the entire contract). (C) is incorrect because while it is true that a contract for the sale of goods for a price of $500 or more must ordinarily be evidenced by a signed writing, the "specially manufactured goods" exception (see above) applies here. (D) is incorrect because the parol evidence rule bars admissibility of evidence that varies an integrated writing; here, there is no writing at all.

A doll collector knew that an acquaintance from her doll collectors' club coveted one particular doll that she owned. The doll collector mailed a letter to the acquaintance on May 3 offering to sell the doll to her for $750. Her letter arrived on May 4. On May 5, the doll collector changed her mind and immediately mailed a revocation to the acquaintance. This revocation arrived on May 7. As the mail carrier handed it to her, the acquaintance simultaneously handed to the mail carrier her own letter to the doll collector, unequivocally accepting her offer. What is the result of the actions here? response - correct AThe revocation was effective upon mailing, and the acceptance would be treated as a counteroffer. BThe acceptance was effective, as long as the acquaintance had no knowledge of the contents of the doll collector's letter when she handed her letter to the mail carrier. Correct CThe outcome would turn on the court's determination as to whether the doll collector's letter had been received by the acquaintance before she had entrusted the letter of acceptance to the mail carrier. DHanding a letter to a mail carrier is not a proper posting of the acceptance, and hence the acquaintance's purported acceptance is not timely.

The outcome would turn on the court's determination as to whether the doll collector's letter had been received by the acquaintance before she had entrusted the letter of acceptance to the mail carrier. At common law, an acceptance is effective upon dispatch (e.g., upon mailing a properly addressed and stamped letter) under the mailbox rule. The mailbox rule does not apply to revocations, however-revocations are effective only upon receipt. Receipt does not require knowledge of the revocation, but merely possession of it. The communication need not be read by the recipient to be effective. [See Restatement (Second) of Contracts §68] The facts here present a close question as to whether there has been a dispatch of the acceptance before the receipt of the revocation. The outcome of this question will depend on the court's determination as to what came first (the posting of the acceptance or receipt of the revocation). This will decide the existence or nonexistence of the contract. (A) is incorrect because, as indicated above, revocation is effective only upon receipt, not mailing. (B) is incorrect because whether the acceptance is effective depends on whether the revocation was received before the acceptance was dispatched, and whether the revocation was received first is not dependent on whether the acquaintance had knowledge of its contents, but rather it depends on whether she had possession of it. (D) is incorrect because the mailbox rule makes an acceptance effective upon posting, and there is no reason to hold that handing a properly addressed, stamped letter to a mail carrier is not a valid posting.

A small business owner decided to retire, so she offered her long-time employee a chance to buy the business for $1 million. She promised in writing to keep the offer open to him for 90 days and to give him enough time to secure financing once he accepted the offer. Over the next few days, the employee cashed out all his retirement accounts and took a second mortgage on his home to raise the funds to purchase the business. When he approached the business owner to discuss the details of the sale, she said that she changed her mind and was revoking her offer because she did not want to retire after all. Was the owner's revocation of her offer proper? response - correct Correct AYes, because it was an offer that could be revoked at will. BNo, because the owner created an option contract by promising to keep the offer open for 90 days. CNo, because the employee detrimentally relied on the offer. DNo, because the offer constitutes a merchant's firm offer.

The owner's revocation of her offer was proper because the offer could be revoked at will. Generally, offers can be revoked at will by the offeror, even if she has promised not to revoke for a certain period of time. There are limitations on the offeror's power to revoke, but none of those exceptions apply in this case. (B) is incorrect because an option contract requires that the offeree give consideration for the promise by the offeror to keep the offer open, and no consideration is indicated by the facts. (C) is also incorrect. Detrimental reliance can limit an offeror's power to revoke where the offeror could reasonably expect that the offeree would rely to his detriment on the offer, and the offeree does so rely. However, this usually is limited to those situations in which the offeror would reasonably contemplate reliance by the offeree in using the offer before it is accepted; e.g., when a general contractor uses a subcontractor's bid in making its own offer. Here, the offer itself included a promise by the owner to give the employee time to secure financing after the offer was accepted. Therefore, the owner had no reason to anticipate that the employee would take immediate steps to raise the purchase money before he even accepted the offer. (D) is incorrect because these facts are not an example of a merchant's firm offer. A merchant's firm offer does not apply to any offer by a merchant; it applies only to an offer under the UCC for the sale of goods where a signed writing gives assurances that the offer will be held open.

A woman entered into a contract with a famous painter for a portrait of her cat. Tragically, the painter, the cat, and the half-completed portrait perished together in a fire. If the woman sues the painter's estate for breach of contract, a court will likely find that the fire caused the painter's contractual duties to be discharged by: response - correct A Frustration Correct B Impossibility C Impracticability D Destruction of the subject matter

The painter's contractual duties will be discharged by impossibility due to the death of the painter. The death of a person necessary to effectuate the contract serves to discharge it for impossibility. Personal service contracts are discharged in this manner only if the services involved are unique. If the services are the kind that can be delegated, the contract is not discharged by the death of the person who was to perform them. Here the woman contracted with a famous painter for the portrait of her cat. His artistic skills would be considered unique, and his death would serve to discharge the contract. This is not a case for discharge by impracticability. The test for a finding of impracticability is that the party to perform has encountered extreme and unreasonable difficulty and/or expense, and its nonoccurrence was a basic assumption of the parties. As discussed above, due to painter's death, his unique performance is now impossible, not merely more difficult or more expensive. The contract was not discharged due to destruction of the subject matter. The cat's death is not destruction of the subject matter. The painting, not the cat, is the subject matter of the contract. If the painter had lived, he still could have painted the cat from memory or from a prior photograph of the cat. Although the painting itself was destroyed as a work in progress, if the painter had survived, he would still be expected to deliver a portrait of the cat, although he may have been given additional time to perform due to the fire. This is similar to when a building in progress is destroyed by an act of nature before completion; the contractor's duty to build is not discharged, but he may be given additional time to perform. This was not a case of frustration. Frustration will exist if the purpose of the contract has become valueless by virtue of some supervening event not the fault of the party seeking discharge. Although the cat has now died, this does not mean that the woman would no longer want the portrait. In fact, she may want the portrait even more as a remembrance of her pet.

The owner of a stationary bicycle wrote a letter to her friend offering to sell her stationary bicycle to him for $150. The friend received the letter on January 18. On January 19, he mailed a letter back saying that he was not interested in purchasing the bike because he had just purchased a gym membership. However, the friend changed his mind the next day and mailed a letter to the owner accepting her offer to sell the bicycle and enclosing a certified check for $150. The owner received the friend's rejection letter on January 21 but put it aside without reading it. The next day, she received the friend's acceptance letter, which she opened and read immediately. Do the parties have a contract? response - correct AYes, because under the mailbox rule an acceptance is effective on dispatch, while a rejection is effective on receipt. BYes, because the friend paid for the bicycle when he accepted the offer to buy it. CNo, because the acceptance was dispatched after the rejection. Correct DNo, because the mailbox rule does not apply-whichever is received first controls.

The parties do not have a contract, because the mailbox rule does not apply when the offeree sends a rejection, followed by an acceptance. In such a case, whichever is received first controls. Under the mailbox rule, acceptance by mail or similar means creates a contract at the moment of posting, with a couple of exceptions not relevant here. Rejection, on the other hand, is effective when received. So, if the mailbox rule had applied, there would have been a contract, because the friend's acceptance was mailed before his rejection letter was received. But because the mailbox rule does not apply here, and the matter is decided based on which letter was received first, there is no contract, because the friend's rejection letter was received by the bicycle owner a day before his acceptance letter was received by her. (A) is incorrect because, as discussed above, the mailbox rule does not apply when a rejection is sent before an acceptance; rather, whichever is received first controls. The fact that the bicycle owner did not read the rejection does not matter; it still was received by her before the acceptance. [See Restatement (Second) Contracts §68] (B) is incorrect because whether the friend paid for the bicycle is irrelevant. He sent the certified check (and his acceptance) after he sent his rejection, and the rejection was received first. (C) is incorrect because when a rejection by mail is followed by an acceptance by mail, the rule is that whichever is received first controls, not whichever is dispatched first. Thus, although it is true that there is no contract between the parties, it is because the friend's rejection letter was received by the bicycle owner first, rather than because it was mailed first.

A professional baseball player visited a sick boy in the hospital. The player told the boy that in consideration of the boy's courage, he would hit a home run for him in his next game. As the player was leaving the hospital, the boy's father stopped the player and told him how important the home run could be in improving his son's spirits and health. The father told the player he would pay him $5,000 if he did hit a home run in his next game. The player agreed and took extra batting practice before his next game to improve his chances. In his next game, the player hit two home runs. The player's contract with his ball club does not forbid him from accepting money from fans for good performance. The player has now asked the father for the $5,000. If the father refuses to pay and the baseball player brings an action against him for damages, which of the following is correct under the prevailing modern rule in contract law? response - correct Correct AThe player can recover the $5,000 because the preexisting duty rule does not apply where the duty is owed to a third person. BThe player can recover the $5,000 if he can prove that the value of the home run to the boy is at least $5,000. CThe player cannot recover from the father because the player had a preexisting duty to use his best efforts to hit home runs. DThe player cannot recover from the father because, even under the modern trend, moral consideration is not valid.

The player can recover because, under the prevailing modern rule, the preexisting duty rule does not apply if the duty is owed to a third person. Generally, contracts must be supported by consideration. A promise to perform is valid consideration, but if a person already owes a duty to perform, traditionally that performance cannot be used as consideration for another promise. Thus, under the traditional rule, the player could not enforce the father's promise to pay the player $5,000 if he hit a home run because the player gave no valid consideration in exchange for the father's promise, since the player owed a preexisting duty to his ball club to exert his best efforts to hit home runs. However, under the modern view as formulated in Restatement (Second) of Contracts, section 73, and followed by a majority of courts, a duty is a preexisting duty only if it is owed to the promisee. Thus, a promise to perform a duty is valid consideration as long as the duty of performance is not already owed to the promisee. In other words, if the duty is owed to a third party, a promise to perform given to another is valid consideration as long as it was bargained for. (B) is incorrect because there is no exception to the preexisting duty rule-modern or otherwise-that allows the promisor to recover merely because his performance benefited a third party. The player can recover under the modern approach because his promise to the father was bargained for. Conversely, the player does not have to prove that the value of his home run to the boy was at least $5,000, because courts generally will not inquire into the adequacy of consideration. (C) would be correct under the traditional rule, but, under the modern trend, the promise here is valid consideration because the duty to hit home runs was owed to a third party (the ball club) rather than to the promisee (the father). (D) is incorrect because while it is true that moral consideration is not good consideration, the father did not rely on moral consideration, but rather exchanged a promise to pay $5,000 for the player's performance.

On July 1, a cattle breeder, who was planning to retire soon, sent a note to his neighbor offering to sell his prize bull for $15,000. On July 10, the neighbor, who was also a cattle breeder, wrote the following note to the retiring breeder: "I have decided to take the bull. I will give you a cashier's check on delivery on Saturday, July 28." The retiring breeder did not respond. The retiring breeder did not want to deliver the bull on July 28 and did not think that the delivery day was agreed to. Instead, he delivered the bull on Monday, July 30. The neighbor refused the delivery and stated that he had found another bull he likes better. The retiring breeder sues the neighbor for breach of contract. Is the retiring breeder likely to prevail? response - correct AYes, because his breach, if any, was minor. BYes, because the parties had not agreed on July 28 as the delivery date. CNo, because there was no contract. Correct DNo, because he did not deliver the bull on July 28.

The retiring breeder will not prevail because he did not deliver the bull on July 28. This is a contract for a sale of goods and thus is governed by the UCC. Under the UCC, an acceptance with additional terms does not constitute a rejection and counteroffer, but rather is an effective acceptance unless made expressly conditional on the assent to the additional terms. Here, the neighbor accepted the offer and added the additional term of a delivery date. Thus, there was a contract. Whether additional terms become part of the agreement depends on whether both parties are merchants. If both parties to the contract are merchants, additional terms in the acceptance will be included in the contract unless they materially alter the terms of the offer, the offer expressly limited the acceptance to its terms, or they are objected to within a reasonable time. Here, both parties are breeders in the cattle business and, thus, are merchants. The change in the delivery date does not materially change the offer (i.e., it does not change a party's risk or remedies), the offer did not limit the acceptance to its terms, and the retiring breeder did not object. Therefore, the July 28 delivery date became part of the contract. By delivering the bull on July 30th, the retiring breeder breached the contract. (A) is incorrect because this is a contract for the sale of goods, which requires perfect tender. Whether the breach was material or minor has no effect. (B) is incorrect because under the UCC, the July 28 term became part of the contract when the breeder failed to object to it. (C) is incorrect because, under the UCC, an acceptance is effective even if it includes additional terms. Thus, the neighbor's letter on July 10 was sufficient to create a contract.

One Saturday, the owner of an art gallery and her friend were discussing art after the friend had helped the owner move some furniture in her home. The friend mentioned that he was very fond of a particular artist. The gallery owner asked her friend if he would like to buy a painting by the artist, entitled "Tears of a Clown," recently consigned to the gallery. The friend said that he would love it, but he only had $2,700. The gallery owner told her friend that she would let him have the painting for that price. The friend knew that the painting was priced at $7,000. He immediately wrote out a check for $2,700 and gave it to the gallery owner, who told him to visit the gallery on Monday to pick up the painting. On Sunday, a salesperson at the gallery sold "Tears of a Clown" to a gallery customer. Neither the salesperson nor the customer knew of the agreement between the gallery owner and her friend. The customer took the painting with him on Sunday. When the friend arrived at the gallery on Monday, the painting was gone. Can the friend obtain specific performance from the gallery owner? response - correct AYes, because there was a bargained-for exchange of promises between the friend and the gallery owner. BYes, because the friend's assistance to the gallery owner in moving her furniture should be considered part of the quantum of adequate consideration. Correct CNo, because the painting was sold to a bona fide purchaser for value and enforcement against the gallery owner is no longer feasible. DNo, because the gallery owner's promise was essentially a gift to her friend that she was free to revoke.

The salesperson sold the painting in good faith to a customer. Because the gallery owner no longer actually has the painting, there is no way to specifically enforce her agreement to convey it to her friend. Specific performance is granted when: (i) there is a valid contract; (ii) the legal remedy is inadequate; (iii) enforcement is feasible; and (iv) mutuality of remedy is present. The gallery owner and her friend had a contract, pursuant to which the gallery owner promised to sell her friend the painting for $2,700. Although this was an oral contract for the sale of goods for a price exceeding $500 and thus subject to the Statute of Frauds, the contract is removed from the Statute by the fact that the friend tendered full payment for the painting. Thus, the oral nature of the agreement is no hindrance to its validity. Moreover, a painting, by its nature, is unique, rendering the legal remedy (damages) inadequate. However, feasibility of specific performance against the gallery owner is lacking here. The salesperson sold the painting to a customer who paid value for it and was unaware that the gallery owner had already agreed to sell it to her friend. The salesperson also was unaware of the gallery owner's agreement with her friend. With the subject matter of the contract having been transferred in good faith to a third party, there is no feasible means to enforce against the gallery owner her agreement to sell the painting to her friend. Thus, the right to specific performance is cut off. Regarding (A), it is true that there was a bargained-for exchange of promises between the gallery owner and her friend. Nevertheless, specific performance is unavailable because enforcement is not feasible. (B) is incorrect because the friend's assistance to the gallery owner in moving her furniture does not form part of the basis of the consideration. The assistance given by the friend occurred before the gallery owner's promise to sell the painting, and thus was not given in exchange for the promise when made. Also, even if the assistance given did form part of the quantum of adequate consideration, specific performance would still be denied because enforcement is not feasible. (D) is incorrect because a gift is a voluntary transfer of property from one person to another without compensation or consideration. The gallery owner clearly stated that she wanted her friend to pay her $2,700 for the painting. Thus, the donative intent necessary for a gift was absent.

Generally speaking, the promise to perform an existing legal duty is __________. response - correct A Past consideration Correct B Not consideration C Sufficient consideration D Valuable consideration

Traditionally the promise to perform, or the performance of, an existing legal duty is not consideration. A promise to perform an existing legal duty is not valuable consideration, unless an exception to the preexisting legal duty rule applies, e.g., new or different consideration is promised, or a minor's ratification of a voidable contract upon reaching the age of majority. Past consideration, which is also not sufficient consideration, is based on something already given or performed, not a promise to perform based on a preexisting legal duty.

On August 1, the owner of a hardware store noticed that he was running low on half-inch carriage bolts and their corresponding nuts. He called a screw manufacturer and ordered 1,000 half-inch carriage bolts and nuts to be delivered by August 15. The screw manufacturer e-mailed the store owner a confirmation of the order that same day. On August 15, the 1,000 bolts were delivered, but the nuts were missing. The store owner called the manufacturer and was told that they had been temporarily out of nuts when they had filled his order, and had reduced the amount he owed to reflect this, as they had done in the past with him in similar circumstances. The store owner protested and the manufacturer offered to send the nuts by overnight carrier so that he would get them the next day. May the store owner cancel the contract? response - correct AYes, because he was entitled to a perfect tender. BYes, because the time for performance has passed. CNo, because the one-day delay is not material. Correct DNo, because the manufacturer has a reasonable amount of time within which to cure.

The store owner may not cancel the contract, because the manufacturer has a reasonable amount of time within which to cure. The general rule in contracts for the sale of goods under the UCC is that the buyer is entitled to a perfect tender, which means that the goods and their delivery must conform exactly to the contract. A few exceptions to this rule exist, including where the seller has reason to think that nonconforming goods will be acceptable to the buyer, which reason can arise from the parties' past dealings. In such a case, upon notification of its intention to cure, the seller must be given a reasonable time within which to cure, which may extend beyond the original time for performance. Here, the manufacturer had reason to think the bolt-only delivery would be acceptable based on the parties' past dealings. Thus, the manufacturer had a reasonable time to cure, and its offer to send the store owner the nuts by overnight carrier is a reasonable offer to cure, negating the store owner's right to cancel the contract. Thus, (A) and (B) are wrong. (C) is wrong because, but for the above exception, the perfect tender rule would apply and the day's delay would be fatal.

A downtown department store engaged an electrician to service all electrical appliances sold by the store for a flat fee of $5,000 per month. Under a written contract signed by both parties, the store was responsible for pickup and delivery of the appliances to be repaired and the billing for the work. By its terms, the contract would continue until either party gave 180 days' written notice of its intent to terminate. Several months ago the electrician informed the store that he was losing money on the deal and was in financial trouble. He requested in good faith that the fee for the next three months be increased by $1,000 and that this increase be paid to a local bank to help pay off a loan that the bank had made to the electrician. The store orally agreed to so modify the original contract. However, the store did not pay the bank and now the bank is suing the store for $3,000. Who will prevail? response - correct Correct AThe store, because there was no consideration to support the promise to pay the bank. BThe store, because the bank is only an incidental beneficiary of the modified contract between the store and the electrician. CThe bank, because it is an intended creditor beneficiary of the modified contract between the store and the electrician. DThe bank, because the electrician exercised good faith in requesting the modification regarding the payment to the bank.

The store will prevail, because there was no consideration to support its promise to pay the bank the additional $1,000 per month. This question looks like it concerns third-party beneficiaries, but it actually presents a consideration issue. Generally, there must be consideration for modification of a contract, and a promise to perform an act that a party is already obliged to do is not sufficient consideration (the "preexisting legal duty" rule). Here, the electrician is promising to do exactly what he was obliged to do under his original contract with the store; thus, there is no consideration to support the promise to increase the fee. Note that the modern view permits modification without consideration if it is fair and equitable in view of unanticipated circumstances. That is not applicable here. This exception contemplates an unanticipated circumstance arising in performance of the contract that makes performance more difficult or expensive. (B) is wrong because the bank is an intended beneficiary, not an incidental beneficiary. An intended beneficiary is one who is clearly intended to benefit from the agreement. Here, the bank was named in the agreement and performance was to be made directly to it, and so it is clearly an intended beneficiary. (C) is wrong even though it is true that the bank is an intended creditor beneficiary. Despite this status, the bank will not recover because there was no consideration to support the modification of the contract. The status of creditor beneficiary does not give the bank any more rights than the electrician would have had to enforce the agreement, and the electrician could not enforce the agreement for the additional money because there was no consideration. (D) is wrong because it is based on the rule of UCC section 2-209, which states that an agreement subject to the UCC does not need consideration to be binding. However, the UCC governs only in cases of the sale of goods, and this question presents a contract for services. Thus, the UCC does not apply and the common law rule requiring consideration controls.

A building that is the subject of a contract between its owner and a contractor is completely destroyed by an act of nature. If the contractor was working on a renovation, the destruction _____________; if the contractor was constructing the building, the destruction ________________. response - correct A Does not discharge the contractor's duties; discharges the contractor's duties by impossibility Correct B Discharges the contractor's duties by impossibility; does not discharge the contractor's duties C Discharges the contractor's duties by impracticability; discharges the contractor's duties by frustration D Discharges the contractor's duties by frustration; discharges the contractor's duties by impossibility

The total destruction by an act of nature of a renovation in progress discharges the contractor's duties by impossibility. If a contract's subject matter is destroyed without the fault of either party, the contractual duties are discharged. If the original building no longer exists, it is impossible to renovate it. The construction of a new building, even if destroyed during progress, is not impossible and thus will not discharge the contractor's duty to perform. However, if the destruction was not caused by the contractor, courts typically will extend the time for the contractor to perform. Discharge by impracticability occurs when performance is possible, but can be accomplished only with extreme and unreasonable difficulty or expense. It is impossible, not impracticable, to renovate a building that no longer exists. Moreover, discharge by frustration may not be raised by the contractor in either case. His purpose in entering into the contract was to make money. Frustration occurs when the purpose of the contract has become valueless by virtue of a supervening event. Frustration of purpose would be a valid defense of the owner in the contract for renovation. Without the building, there is no point in paying to renovate it.

At common law, the Statute of Frauds requires _____________ signed by ____________. response - correct Correct A A writing or writings reflecting the material terms of the contract; the party to be held liable B A formal contract; the party to be held liable C A formal contract; both parties D A writing or writings reflecting the material terms of the contract; both parties

To satisfy the Statute of Frauds, there must be one or more writings that reflect the material terms of the contract signed by the person sought to be held liable on the contract. The Statute does not require both parties to sign, only the party to be charged. The Statute of Frauds does not require a formal written contract or the signature of both parties. For example, a letter, receipt, or a check containing the material terms (e.g., quantity for sale of goods) and signed by the party to be charged satisfies the Statute of Frauds.

Which of the following is required for a merchant's firm offer under Article 2? response - correct A Consideration paid by the offeree to keep the offer open Correct B A written assurance signed by the offeror C Both the offeree and offeror are merchants D A specific time frame that the offer will remain open

Under Article 2, a merchant's firm offer arises when a merchant offers to buy or sell goods in a signed writing and the writing gives assurances that the offer will be held open. If no specific time frame is stated in the offer, a merchant's firm offer will remain open for a reasonable time (but in no event may such period exceed three months). For a merchant's firm offer, it is not necessary that both parties be merchants; only the offeror must be a merchant. A merchant's firm offer is enforceable even if no consideration has been paid by the offeree to keep the offer open.

A merchant who offers to buy or sell goods in a signed writing that gives assurances that the offer will be held open is offering: response - correct A An option contract Correct B A merchant's firm offer C A unilateral contract D A confirmatory memo contract

Under Article 2, a merchant's firm offer arises when a merchant offers to buy or sell goods in a signed writing that gives assurances that the offer will be held open. If no specific time frame is stated in the offer, a merchant's firm offer will remain open for a reasonable time (but in no event may such period exceed three months). An option contract is a distinct contract in which the offeree gives consideration for a promise by the offeror not to revoke an outstanding offer for a period of time. An offer for a unilateral contract is one that can be accepted only by full performance. Note that the beginning of performance may create an option so that the offer is irrevocable. However, the offeree is not obligated to complete performance merely because he has begun performance, as only complete performance constitutes an acceptance of the offer. A confirmatory memo is not an offer. It is a method of satisfying the Statute of Frauds in contracts between merchants. The confirmatory memo rule states that if one party, within a reasonable time after an oral agreement has been made, sends to the other party a written confirmation of the understanding that is sufficient under the Statute of Frauds to bind the sender, it will also bind the recipient if: (i) he has reason to know of the confirmation's contents; and (ii) he does not object to it in writing within 10 days of receipt.

Which of the following contracts must be evidenced in writing? response - correct A A contract to build a building Correct B A mortgage contract C A six-month lease of a parcel of land D A contract between business partners to buy and sell real estate and divide the profits

Under the Statute of Frauds, a promise creating an interest in land must be evidenced by a writing. This includes not only agreements for the sale of real property, but also other agreements pertaining to land, such as a mortgage contract. Some contracts may have an end result involving an interest in land, but they still do not come within the Statute. For example, a contract to build a building or a contract to buy and sell real estate and divide the profits do not come within the Statute. A lease of a parcel of land for more than one year is also covered by the Statute, but a six-month lease is not.

A landowner advertised in the newspaper that he wished to sell 40 acres of land at $10,000 per acre. A rancher who was looking to expand his holdings was interested, so he came out to inspect the property. After the inspection, the rancher agreed to purchase the land for $400,000. A contract for the sale of the 40 acres was prepared and signed by the landowner and the rancher. The contract failed to state the purchase price. Later, the rancher had a change of heart and refused to complete the purchase. In the landowner's lawsuit for breach of contract, for which party would the court likely hold? response - correct AThe landowner, because the parol evidence rule will not bar testimony that the rancher agreed to pay $400,000. BThe landowner, because the Statute of Frauds can be satisfied by combining the original advertisement and the written contract. CThe rancher, because the parol evidence rule will bar all evidence that he agreed to pay $400,000 for the land. Correct DThe rancher, because the Statute of Frauds would require the contract to contain the price in order to be enforced.

Under the Statute of Frauds, contracts for the sale of land must be in writing. The writing must contain all essential terms, and the price is considered an essential term. (A) is wrong because although the parol evidence rule might not bar the testimony, the Statute of Frauds will prevent recovery. (B) is wrong; the advertisement was not signed by the rancher, the party charged with breaking the contract. Thus, it is not a memorandum. Furthermore, the ad could not be considered part of the contract because there is nothing in the question indicating that it was attached to or referred to in the contract, or that it was assented to by the parties as part of the contract. In fact, an ad is a mere offer to deal; the actual price term may be very different by the time parties to a contract reach an agreement. (C) is wrong; the parol evidence rule would not bar the testimony, and in any event, that is not the reason the rancher will win.

Which of the following acts alone would be sufficient to allow enforcement of an oral contract for the sale of real property? response - correct A Possession of the property by the purchaser B Payment of the full purchase price by the purchaser C Valuable improvements to the property by the purchaser Correct D Conveyance of the property from the seller to the purchaser

Upon the seller's conveyance of the property to the purchaser, the seller can enforce the buyer's oral promise to pay. Under the doctrine of part performance, conduct that unequivocally indicates that the parties have contracted for the sale of the land will take the contract out of the Statute of Frauds. However, most jurisdictions require at least two of the following: payment (in whole or in part), possession, and/or valuable improvements.

When the amount due on a debt is undisputed, which of the following will not be considered sufficient consideration for a promise by the creditor to discharge the debt? response - correct A Payment in a different medium. B Payment to one other than the creditor. Correct C Payment of a smaller sum than due. D Payment before maturity.

When the amount due is undisputed, payment of a smaller sum than due will not be sufficient consideration for a promise by the creditor to discharge the debt. Neither a legal detriment nor a benefit would be present. In contrast, if the consideration is in any way new or different, such as payment before maturity or to one other than the creditor; or payment in a different medium (e.g., stock instead of cash), then sufficient consideration may be found.

Which of the following would serve as sufficient consideration for a promise by a creditor to discharge an existing debt? response - incorrect Incorrect A Unforeseen difficulty in performance by the debtor Correct B An alternative method of payment C Partial payment of the debt D Acknowledgement of the existence of the debt

When the proposed consideration is in any way new or different (e.g., an alternative method of payment), there is usually sufficient consideration to change a preexisting duty, such as discharging an existing debt. Mere acknowledgment of a preexisting duty is not sufficient consideration to change the preexisting duty. Partial payment of the amount due on an existing debt is not sufficient consideration for a promise by the creditor to discharge the debt. Neither a legal detriment nor benefit is present. Under the majority view, mere unforeseen difficulty in performance is not a substitute for consideration. Although the modern view permits modification without consideration if the modification is fair and equitable in view of circumstances not anticipated when the contract was made, it would not apply to payment of an existing debt. That exception to the consideration requirement applies only if the contract has not been fully performed on either side, and an existing debt suggests that the creditor has already performed. Also, as with impracticability, difficulty in paying money would be unlikely to be considered the type of unforeseen circumstance this view is intended to address.

Under the Article 2 battle of the forms provision, whether additional or different terms proposed by the offeree during acceptance ultimately become part of a contract depends on whether or not __________. response - correct A The offeree is a merchant Correct B Both parties are merchants C The offeror is a merchant D One of the parties is a merchant

Whether the additional or different terms become part of the contract depends on whether or not both parties are merchants. If any party to the contract is not a merchant, the additional or different terms are considered to be mere proposals to modify the contract. They do not become part of the contract unless the offeror expressly agrees. If both parties are merchants, additional terms in the acceptance become part of the contract unless they materially alter the terms of the offer, the offer expressly limits acceptance to the terms of the offer, or the offeror has already objected to the terms (or objects within a reasonable time after notice of them is received). Between merchants, some courts treat different terms in an acceptance the same as additional terms; other courts apply the knockout rule (i.e., conflicting terms are knocked out and replaced by gap-filling terms under the UCC).


Ensembles d'études connexes

The Purchasing Process - Ch 12 - AC 304

View Set

docs.aws.amazon - Migration Planning 15%

View Set

Inventory and Cost of Goods Sold

View Set

Learn Smart PKA- Chapter Six: An Introduction to Energy, Enzymes, and Metabolism

View Set

Test Pharmacology Made Easy 4.0 - Endocrine System

View Set

Consumer Behavior: Chapter 16 - Alternative Evaluation and Selection

View Set

Cláusulas nominales, desencadenantes

View Set

ATI: RN Nursing Care of Children Online Practice 2019 A with NGN

View Set

Cervical and Thoracic Spine (Ch 8)

View Set